You are on page 1of 52

FORM 26 PART 1

y
1. A 22-year-old man is brought to the emergency department 20 minutes after sustaining a knife wound to the right side of his chest in a fight at a local bar. On arrival, he is
short of breath. His pulse is 108/min, respirations are 30/min, and blood pressure is 100/60 mm Hg. Physical examination shows that the trachea is deviated to the left.
Further physical examination of the traumatized side is most likely to show which of the following pulmonary findings?

O A ) Crackles
B ) Egophony
O C ) Hyperresonant percussion
O D ) Increased fremitus
O E ) Stridor
® F ) Wheezing
O G ) Whispered pectoriloquy
Correct Answer: C.

2. A 9-year-old boy is brought to the physician because he told his teacher that his fingers felt "funny" after he swung
on the monkey bars 2 hours ago. Neurologic examination shows numbness and tingling in the ring and small
fingers of the left hand. The physician concludes that the ulnar nerve may have been stretched while he was
hanging from the bars. Which of the following other functions should be checked to assess the integrity of the ulnar
nerve in the left limb?

® A ) Abduction of the index, middle, ring, and small fingers


O B ) Abduction of the thumb
C ) Extension of the index, middle, ring, and small fingers
O D ) Extension of the thumb
O E ) Opposition of the thumb
Correct Answer: A.

y
3. An investigator is studying patients with West Nile virus infection. During a 5-year period, data are collected on 25 patients
diagnosed with this condition as confirmed by testing at the Centers for Disease Control and Prevention. Demographic
information on the patients is reported (age, gender, and ethnicity), as well as information about likely sources of infection. Which
of the following best describes this study design?

O A ) Case series
B ) Case-control study
® C ) Cohort study
D ) Correlational study
E ) Cross-sectional study
Correct Answer: A.

Y 4. A 55-year-old woman who is a physician is admitted to the hospital because of a fractured femur sustained in a motor vehicle
collision. Two days after admission, she develops tachycardia, restlessness, diaphoresis, and anxiety. She says that "vague
shapes" are coming out of the walls. Which of the following is the most likely cause of this patient's current condition?

A ) Acute stress disorder


O B ) Alcohol withdrawal
C ) Bipolar disorder
D ) Histrionic personality disorder
® E ) Panic disorder
Correct Answer: B.
y
5. A newborn is delivered at 40 weeks' gestation to a 20-year-old woman after an uncomplicated pregnancy. Physical examination
shows male-like external genitalia, with ditoral hypertrophy and labial fusion. The karyotype is 46,XX . The serum concentration
of cortisol is decreased and serum concentrations of 17-hydroxyprogesterone, androstenedione, and testosterone are increased.
Which of the following is the most likely cause of the findings in this newborn?

® A ) Androgen receptor mutation


O B ) Defect in adrenal steroid biosynthesis
C ) Deficiency of enzyme activity in gonadal steroid hormone biosynthesis
D ) Increased anti-mullerian hormone secretion
E ) Translocation of the SRY gene to one of the X chromosomes
Correct Answer: B.

6. A 35-year-old woman, gravida 3, para 2, develops a massive hemorrhage after the vaginal delivery of a healthy female newborn
at 38 weeks' gestation. She underwent a cesarean delivery 2 years ago because of cephalopelvic disproportion. A hysterectomy
is required at this time to control the bleeding. The gross and microscopic appearances of the uterus are shown. Which of the
following is the most likely cause of the postpartum hemorrhage in this patient?

® A ) Abruptio placentae
B ) Ectopic pregnancy
C ) Endometriosis
D ) Leiomyomata uteri
O E ) Placenta accreta
F ) Placental site trophoblastic tumor
Correct Answer: E.

7. A 70-year-old man with poorly controlled hypertension is brought to the emergency department 30 minutes after the sudden
onset of weakness of the left side of his face and body. He is alert and able to follow commands. His pulse is 80/min, and blood
pressure is 190/90 mm Hg. Physical examination shows mild flattening of the nasolabial fold on the left side and weakness and
hyperreflexia of the left upper and lower extremities. There are no sensory deficits, but he has mild dysarthria. Six months later,
histologic examination of the area of the injury is most likely to show proliferation of which of the following cell types?

® A ) Astrocytes
B ) Ependymal cells
C ) Microglial cells
D ) Neurons
E ) Oligodendrocytes
Correct Answer: A.

y
8. An 18-month-old boy in West Africa with perinatally acquired HIV infection is exposed to a child in his village with a fever and an
erythematous, maculopapular rash. Three weeks later, the 18-month-old boy develops giant cell pneumonia, but there is no
evidence of a rash. Which of the following viruses is the most likely cause of the pneumonia?

A ) Coxsackievirus
B ) Influenza virus
C ) Measles virus
® D ) Rubella virus
E ) West Nile virus
Correct Answer: C.
9. A previously healthy 19-year-old woman comes to student health services because of a 3-day history of vulvar itching and
vaginal discharge. She has a new sexual partner and uses no contraception. Speculum examination shows a profuse yellow-
gray discharge in the vagina. The pH of the discharge is 6. Microscopic examination of the discharge shows clue cells. Addition
of KOH to the discharge produces a strong amine odor. Which of the following is the most likely cause?

® A ) Bacterial vaginosis
B ) Bacteroides fragilis
C ) Candida albicans
D ) Haemophilus ducreyi
E ) Herpes simplex virus
O F ) HIV
G ) Human papillomavirus
H ) Pneumocystis jirovecii (formerly P. carinii )
I ) Treponema pallidum
J ) Trichomonas vaginalis
Correct Answer: A.

y
10.. Afemale
A female newborn is delivered vaginally at 37 weeks' gestation after an uncomplicated pregnancy, labor, and delivery. Physical
examination shows a narrow cranium that is elongated in the anteroposterior dimension. Premature closure of which of the
following during embryogenesis is the most likely cause of this anomaly?

A ) Bregma
B ) Coronal suture
C ) Lambda
® D ) Lambdoid suture
O E ) Sagittal suture
Correct Answer: E.

y
Y 11. A healthy 35-year-old woman who wishes to become a regular platelet donor comes to a blood donation center. Blood is
obtained by venous phlebotomy into sodium citrate and processed to yield platelet-rich plasma (PRP) for platelet function
studies. She has no personal or family history of bleeding disorders. Aliquots of the PRP are subsequently analyzed using a
light-transmittance platelet aggregometer. Adding a 5 pM solution of which of the following reagents to the PRP is most likely to
cause a rapid and irreversible aggregation pattern in these studies?

A ) Adenosine diphosphate
® B ) Collagen monomer
C ) Immune globulin
D ) Norepinephrine
E ) Prostacyclin (PGIj
PGI 2)
Correct Answer: A.

y
12. A 64-year-old woman develops pain and stiffness of the proximal interphalangeal joints and the right knee. The pain is made
worse by activity and is relieved by rest. X-rays of the knee show a narrowed joint space with radiodense subchondral bone
and cyst formation. Knee fluid aspirate shows:
Appearance clear, yellowish
Leukocyte count 250/mm 3 (N<200)
Neutrophils 5% (N<25%)
Glucose 101 mg/dL (N=80-100)

Which of the following is the most likely cause of her condition?

A ) Acute gouty arthritis


O B ) Osteoarthritis
C ) Rheumatoid arthritis
®
® D ) Septic arthritis
E ) Normal age-related changes in joints
Correct Answer: B.
y
13. An 8-year-old boy is brought to the physician by his parents because of disruptive behavior. They say, "He is easily distracted,
constantly interrupts us while we are talking, and seems to be always moving. His teacher says he is always talking with his
neighbors, has trouble completing tasks, and refuses to wait his turn when playing games." Physical examination shows no
abnormalities. If drug therapy is indicated, administration of a drug with which of the following mechanisms of action is most
appropriate?

® A ) Antagonism at (3-adrenergic receptors


B ) Blockade of voltage-gated Na + channels
C ) Enhanced action of y-aminobutyric acid (GABA) at GABAA receptors
O D ) Increased release of dopamine and norepinephrine
E ) Selective inhibition of uptake of serotonin at nerve terminals
Correct Answer: D.

Y
14. A 62-year-old man undergoes a physical examination on his arrival at the
penitentiary to serve his prison sentence. A routine PPD skin test shows a
14-mm area of induration at 48 hours. A chest x-ray is shown. Reciprocal
activation of the two immune cell types involved in the pathogenesis of this
patient’s pulmonary lesions is mediated by interferon-gamma and which of the
following cytokines?

A ) lnterleukin-4 (IL-4)
® B ) IL-5
C ) IL-7
D ) IL-10
O E ) IL-12

Correct Answer: E.

y
15. A 12-year-old boy is admitted to the hospital because of lethargy, hip pain, and a temperature of 39.4° C (103°F). He has been hospitalized several
other times because of pneumonia. His neonatal period was normal. Complete blood counts are within normal limits, and a test for HIV antibody is
negative. Blood cultures grow Staphylococcus aureus. Serum electrophoresis is most likely to show which of the following patterns?

Albumin

® A)
B)
C)
D)
E)
Correct Answer: D.
y
16. A 32-year-old woman comes to the physician 1 day after she noticed a large red lesion on her back. One week ago, she returned from vacation in
Cape Cod. After evaluation, a diagnosis of erythema chronicum migrans is made. Prior to prescribing an antibiotic, the physician asks the patient if
she could possibly be pregnant because an adverse effect of the antibiotic active against the causal organism is incorporation into fetal bones and
teeth, causing yellow discoloration. The most likely mechanism of this drug is inhibition of which of the following?

O A ) Attachment of tRNA to ribosomes


B ) Dihydrofolate reductase
O C ) DNA repair by DNA gyrase
(• D ) Electron transport
O E ) Transpeptidation of peptidoglycan
Correct Answer: A.

17. A female newborn delivered at term develops jaundice, hepatomegaly, and rash shortly after delivery. Pregnancy was complicated by a 2-week
febrile illness during the first trimester. The newborn is at the 25th percentile for length, 30th percentile for weight, and 20th percentile for head
circumference. One week later, a CT scan of the head shows intracranial calcifications. Two days later, she becomes increasingly lethargic and
develops respiratory distress and seizures. She has a 2-year-old sister who attends day care. Which of the following is the most likely causal
virus?

® A ) Cytomegalovirus
B ) Herpes simplex virus
0 C ) Measles virus
D ) Mumps virus
E ) Varicella-zoster virus
Correct Answer: A.

y
18. A previously healthy 35-year-old woman develops hypoxemia 35 minutes after ingesting a near-lethal dose of barbiturates. She has not aspirated.
Which of the following sets of arterial blood gas values (in mm Hg) is most likely in this patient?

Po 2 Pco 2 (A-a)02
O A) 40 50 35
0 B) 40 60 40
® C) 50 25 10
D) 50 80 10
E) 60 35 25
Correct Answer: D.

y
19. A 1-year-old boy is brought to the physician by his parents because of a 4-week history of progressive generalized tonic-clonic seizures and a
strange odor to his urine. He has a history of delayed development. He was adopted from an orphanage in Russia at the age of 6 months. Physical
examination shows fair skin and blond hair. His phenylalanine hydroxylase gene is homozygous for a point mutation (GT — AT) in intron 12 of the
affected gene that causes skipping of exon 12. Which of the following is the most likely explanation for exon skipping in this patient's affected gene?

O A ) Alternative polyadenylation site


B ) Deletion of 5' untranslated region
® C ) Expansion of trinucleotide repeat
D ) Gene duplication
E ) Nonhomologous recombination
F ) Nonsense mutation
O G ) RNA splice error
H ) X inactivation
Correct Answer: G.

y
20. A 65-year-old woman has ascites. Which of the following additional findings indicates a diagnosis of constrictive pericarditis rather than cirrhosis?

0 A ) Edema of the lower extremities


C B ) Esophageal varices
C C ) Hypoalbuminemia
@ D ) Hyponatremia
O E ) Increased jugular venous pressure
O F ) Splenomegaly
Correct Answer: E.
y
21. A 37-year-old man comes to the physician because of a 1-week history of nausea, vomiting, and abdominal pain. He ate raw oysters while
vacationing in Mexico 2 weeks ago. He has smoked 1 pack of cigarettes daily for the past 15 years but has now lost the desire to smoke. Physical
examination shows scleral icterus and slight enlargement and tenderness of the liver. Which of the following pathogens is most likely responsible for
the development of these symptoms in this patient?

O A ) Epstein-Barr virus
O B ) Hepatitis A virus
O C ) Hepatitis B virus
D ) Hepatitis C virus
@ E ) Vibrio vulnificus
Correct Answer: B.

22. Which of the following labeled depolarizations is associated with the smallest stroke volume?

3 A)
® B)
C)
D)
3 E)
Correct Answer: B.

y
23. An investigator conducts a clinical study of 300 patients with stages III and IV colon carcinoma. It is found that over time, metastases show an
increasing potential for growth and resistance to chemotherapy. Which of the following mechanisms of the cell cycle best explains this observation?

0 A ) Decreased expression of cyclin B during G /M transition



( B ) Decreased phosphorylation of RB1 protein during G S transition
^
O C ) Genomic instability during S, G and M phases
* ,
D ) Overexpression of p15, p16, and p18 during G phase
E ) Overexpression of p21, p27, and p57 proteins during all phases
Correct Answer: C.

y
24. A 45-year-old homeless man is brought to the emergency department by police 30 minutes after he was found unconscious. His breath and clothes
smell of alcohol. His temperature is 36.8°C (98.2°F), pulse is 68/min, respirations are 14/min, and blood pressure is 110/55 mm Hg. Physical
examination shows bronzed skin and spider angiomata on the chest. Laboratory studies show:
Hemoglobin 10 g /dL
Hematocrit 30%
Mean corpuscular volume 110 pm 3
Leukocyte count 9000/mm 3
Segmented neutrophils 70%
Lymphocytes 20%
Monocytes 10%
Platelet count 160,000/mm 3
Serum
Ferritin 200 ng/mL
Vitamin B 12 (cobalamin) 500 pg/mL (N=160-950)
RBC folate 20 ng/mL (N=166-640)

A peripheral blood smear shows occasional hypersegmented neutrophils and 3+ oval macrocytes. Serum studies are most likely to show which of
the following sets of additional findings?

Methylmalonic Acid Homocysteine


® A) Increased increased
3 B) Increased normal
3 c) Increased decreased
3 D) Normal increased
E) Normal normal
F) Normal decreased
3 G) Decreased increased
H) Decreased normal
I) Decreased decreased
y
25. A 54-year-old woman with a 25-year history of rheumatoid arthritis that is refractory to therapy is
admitted to the hospital for a joint replacement operation. Preoperative laboratory studies show:
Hemoglobin 10.5 g/dL
Mean corpuscular volume 79 pm 3
Leukocyte count 8900/mm 3
Platelet count 230,000/mm 3 % jt
Serum
Iron 40 pg/dL
Transferrin 220 mg/dL (N=200-400)

A photomicrograph of a Prussian blue iron-stained smear of bone marrow aspirate is shown. Which of
the following is the most likely cause of the anemia?

O A ) Autoimmune hemolysis
O B ) Chronic disease
0 C ) Iron deficiency
D ) Marrow replacement by neoplastic tissue
E ) Myelodysplasia

( ) F ) (3-Thalassemia minor
%
Correct Answer: B.

26. A 40-year-old woman with chronic paraplegia caused by multiple sclerosis is brought to the physician because of severe, debilitating muscle
cramps for the past 3 weeks. Treatment with baclofen resolves her muscle cramps. Which of the following receptors most likely decreased the
muscle spasticity in this patient?

O A ) a 2-Adrenoreceptor
B ) (32-Adrenoreceptor
® C ) y-Aminobutyric acid B receptor
D ) Calcium-sensing receptor
E ) Ryanodine receptor
Correct Answer: C.

27. A 35-year-old man is brought to the emergency department because of multiple injuries sustained in a motor vehicle collision 45 minutes ago. He
is lethargic and confused. The right pupil is sluggishly reactive to light and is 1 mm larger than the left pupil. There is mild left hemiparesis. Further
examination shows fracture of the right femur, ruptured spleen, and hemorrhagic pleural effusion. ACT scan of the head shows a subdural
hematoma on the right with mass effect and midline shift. He is intubated and mechanically hyperventilated. Which of the following is the most
likely rationale for hyperventilation in this patient?

® A ) Cause cerebral vasoconstriction


B ) Decrease pulmonary atelectasis
C ) Increase cerebral blood flow
G D ) Increase intracranial pressure
E ) Increase oxygenation
Correct Answer: A.

y
28. A 45-year-old man comes to the physician because of a 2-month history of a dull pain on the left side of his scrotum. Examination of the left side of
the scrotum shows a bluish appearance of the skin and a palpable mass that feels like a bag of worms. Which of the following veins is most likely
obstructed in this patient?

0 A ) Inferior mesenteric vein


C B ) Inferior vena cava
C ) Inferior vesical vein
5 D ) Internal iliac vein
(

O E ) Left renal vein


Correct Answer: E.
29. After being given an infusion of mannitol (400 mM), a healthy person is most likely to have which of the following changes in plasma osmolality and
plasma ADH (vasopressin) concentration?

101
8
s
-
SE
>
Ia.
UI
5 -

(/>
rc
Q
- 0 -
280 290 300 31C
Plasma osmolality ( mOsmol / L )

O Control before infusion


® A)
B)
O C)
D)
E)
Correct Answer: A.

y
30. A 35-year-old man comes to the physician because of a 6-month history of burning abdominal pain that occurs 1 to 2 hours after he eats. He also
has had black stools for 2 days. Use of over-the-counter antacids and histamine-2 (H -receptor blocking agents has not been effective in relieving

- ^
his symptoms. He is sweating profusely and has light-headedness when he stands. His blood pressure is 105/70 mm Hg while sitting. Physical
examination shows epigastric tenderness. ACT scan of the abdomen shows a 1 cm mass in the pancreas. Immunohistochemical labeling of
neoplastic cells in a biopsy specimen is most likely to involve the use of antibodies directed to which of the following substances?

A ) Amylase
B ) Gastrin
0 C ) Glucagon
D ) Human pancreatic polypeptide
E ) Insulin
0 F ) Lipase
(• G ) Serotonin
H ) Somatostatin
I ) Vasoactive intestinal polypeptide
Correct Answer: B.

y
31. A 52-year-old woman with a long-standing history of generalized anxiety disorder comes to the physician for a follow-up examination. She has been
taking an anxiolytic drug for 12 years, but she is not currently under a psychiatrist's care because she says she is feeling better and that she prefers
this physician to psychiatrists. She has hyperlipidemia, hypertension, and mild type 2 diabetes mellitus treated with pharmacotherapy. Following
physical examination and laboratory studies, the physician makes adjustments to the patient's medication regimen. Two days later, the physician
receives 12 calls from the patient for clarification of treatment recommendations; this pattern has occurred frequently in the past. In particular, the
patient fixates on the potential toxicities of her medications. She often asks questions that have specifically been answered during the recent office
visit. Which of the following is the best approach to manage this patient's needs?

A ) Advise the patient that constant questioning of recommendations suggests a lack of trust, and that she should seek care from a
provider in whom she has confidence
© B ) Advise staff to ignore the patient's calls if they are too frequent
C ) Contact one of the patient's family members to engage his or her help in enforcing more responsible use of the physician's time
O D ) Insist that the patient obtain routine psychiatric care
O E ) Set conditions and establish limits for phone calls, and set up a regular schedule of office visits
Correct Answer: E.

y
32. A 55-year-old man comes to the physician because of a 2-month history of decreased appetite; he also has had a 9-kg (20-lb) weight loss and an
intermittent rash during this period. Physical examination shows necrolytic migratory erythema over the axillae and groin. Laboratory studies show a
serum glucose concentration of 280 mg/dL and plasma glucagon concentration of 1500 pg/mL (N=20-100). ACT scan of the abdomen shows a
mass at the head of the pancreas. Which of the following processes is most likely occurring in this patient's liver?

C A ) Decreased activity of carnitine acetyltransferase


C B ) Glycogen synthesis
C ) Increased activity of acetyl-CoA carboxylase
D ) Inhibition of ketogenesis
O E ) Partial oxidation of fatty acids
Correct Answer: E.
y
33. A health care screening for diabetes mellitus is conducted at a weekend market with subjects including both men and women ranging in age from
25 to 95 years. Blood samples are obtained, and the blood glucose concentrations are determined. The results are shown in the table:
Patient Glucose (mg/dL)
1 70
2 120
3 120
4 140
5 110
6 95
7 75
8 145
9 105
10 85
11 85
12 90
13 120
14 100
15 190

Which of the following best represents the median blood glucose concentration for this group (in mg/dL)?

® A ) 95
B ) 105
C ) 110
D ) 120
E ) 125
Correct Answer: B.

y
34. An investigator is studying a laboratory strain of HIV-1. During the experiment, point mutations are introduced in the long terminal repeat region of
the virus. Infection of a human T-lymphocyte line with the mutant virus results in markedly decreased viral replication compared with infection with
the wild-type virus. A mutation most likely directly affected which of the following phases of the viral life cycle?

C A ) Attachment of the virus to the cell surface


B ) Budding of the virus from the cell membrane
(•) C ) Fusion of the viral envelope and cell membrane
O D ) Integration of the provirus into the host cell genome
E ) RNA splicing to produce an mRNA encoding the envelope glycoproteins
Correct Answer: D.

/ 35. A 56-year-old man is brought to the emergency department 1 hour after having a generalized tonic-clonic seizure during his morning run. Vital
signs are within normal limits. MRIs of the brain show a mass in the lateral aspect of the precentral gyrus of the frontal lobe. Based on the MRI
findings, which of the following additional deficits is most likely in this patient?

O A ) Atrophy of the left upper extremity


B ) Decreased proprioception in the right upper extremity
C ) Decreased sensation in the left lower extremity
D ) Deviation of the tongue to the right
® E ) Weakness of the left lower area of the face
F ) Weakness of the right lower extremity
Correct Answer: E.

/ 36. A male newborn is found to have a defect in anchoring fibrils. Which of the following skin findings is most likely in this patient?

® A ) Blisters
B ) Easy bruising
C ) Eczematous rash
D ) Inability to sweat
E ) Thickened skin
F ) Ulcers
Correct Answer: A.
y
37. A 55-year-old woman with systemic lupus erythematosus has an increased serum C-reactive protein (CRP) concentration. Which of the following is
the most likely source of the increased serum CRP concentration in this patient?

0 A ) Bone marrow
3 B ) Brain
® C ) Kidney
O D ) Liver
3 E ) Lymph node
F ) Skin
Correct Answer: D.
y
38. A 52-year-old woman undergoes surgical excision of a large multinodular goiter. During the operation, the superior thyroid artery is used as a
landmark to avoid damage to a nerve proximal to that vessel. Which of the following best describes this nerve?

0 A ) External branch of the superior laryngeal nerve


B ) Inferior root of the ansa cervicalis
0 C ) Nerve to the mylohyoid
® D ) Phrenic nerve
E ) Transverse cervical nerve
Correct Answer: A.

y
39. A 23-year-old woman is brought to the emergency department because of shortness of breath for 2 weeks. Her respirations are 28/min. Physical
examination shows no other abnormalities. Laboratory studies show:
Serum
Na + 135 mEq/L
K* 4.0 mEq/L
Cl- 110 mEq/L
HCO 3- 15 mEq/L
Arterial pH on room air 7.25
Urine
Sodium 20 mEq/L
Chlorine 30 mEq/L
Potassium 15 mEq/L

Which of the following is the most likely diagnosis?

® A ) Alcoholic ketoacidosis
3 B ) Crohn disease
C ) Diabetic ketoacidosis
3 D ) Lactic acidosis
E ) Renal tubular acidosis
F ) Salicylate poisoning
Correct Answer: E.

y
40. A 40-year-old woman comes to the physician for a follow-up examination 1 week after having a second increased blood pressure measurement. Her
pulse is 72/min, and blood pressure is 168/104 mm Hg in both arms while sitting. Physical examination shows no other abnormalities. Treatment
with lisinopril and hydrochlorothiazide is initiated. One month later, her blood pressure is 120/80 mm Hg. Which of the following sets of hormonal
changes is most likely in this patient?

Angiotensin I Angiotensin II Bradykinin


® A) T t no change
3 B) t l r
3 C) t i i
o) i t T
3 E) 1 t I
Correct Answer: B.

y
41. A 47-year-old man with a history of rheumatic valvular disease comes to the physician because of chest
pain and difficulty breathing for the past 2 days. An ECG shows an acute myocardial infarction.
Examination shows a systolic murmur. Pressure tracings from the aorta, left ventricle (LV), and left atrium
(LA) are shown. Which of the following is the most likely diagnosis? O) Aortic
I
0 A ) Aortic obstruction E
3 B ) Aortic regurgitation £
O C ) Left ventricular aneurysm O)
LVJ
LA
( ) D ) Mitral obstruction
* 3
if)

O E ) Rupture of the chordae tendineae 0)


CL

Time
Correct Answer: E.
y
37. A 55-year-old woman with systemic lupus erythematosus has an increased serum C-reactive protein (CRP) concentration. Which of the following is
the most likely source of the increased serum CRP concentration in this patient?

0 A ) Bone marrow
3 B ) Brain
® C ) Kidney
O D ) Liver
3 E ) Lymph node
F ) Skin
Correct Answer: D.
y
38. A 52-year-old woman undergoes surgical excision of a large multinodular goiter. During the operation, the superior thyroid artery is used as a
landmark to avoid damage to a nerve proximal to that vessel. Which of the following best describes this nerve?

0 A ) External branch of the superior laryngeal nerve


B ) Inferior root of the ansa cervicalis
0 C ) Nerve to the mylohyoid
® D ) Phrenic nerve
E ) Transverse cervical nerve
Correct Answer: A.

y
39. A 23-year-old woman is brought to the emergency department because of shortness of breath for 2 weeks. Her respirations are 28/min. Physical
examination shows no other abnormalities. Laboratory studies show:
Serum
Na + 135 mEq/L
K* 4.0 mEq/L
Cl- 110 mEq/L
HCO 3- 15 mEq/L
Arterial pH on room air 7.25
Urine
Sodium 20 mEq/L
Chlorine 30 mEq/L
Potassium 15 mEq/L

Which of the following is the most likely diagnosis?

® A ) Alcoholic ketoacidosis
3 B ) Crohn disease
C ) Diabetic ketoacidosis
3 D ) Lactic acidosis
E ) Renal tubular acidosis
F ) Salicylate poisoning
Correct Answer: E.

y
40. A 40-year-old woman comes to the physician for a follow-up examination 1 week after having a second increased blood pressure measurement. Her
pulse is 72/min, and blood pressure is 168/104 mm Hg in both arms while sitting. Physical examination shows no other abnormalities. Treatment
with lisinopril and hydrochlorothiazide is initiated. One month later, her blood pressure is 120/80 mm Hg. Which of the following sets of hormonal
changes is most likely in this patient?

Angiotensin I Angiotensin II Bradykinin


® A) T t no change
3 B) t l r
3 C) t i i
o) i t T
3 E) 1 t I
Correct Answer: B.

y
41. A 47-year-old man with a history of rheumatic valvular disease comes to the physician because of chest
pain and difficulty breathing for the past 2 days. An ECG shows an acute myocardial infarction.
Examination shows a systolic murmur. Pressure tracings from the aorta, left ventricle (LV), and left atrium
(LA) are shown. Which of the following is the most likely diagnosis? O) Aortic
I
0 A ) Aortic obstruction E
3 B ) Aortic regurgitation £
O C ) Left ventricular aneurysm O)
LVJ
LA
( ) D ) Mitral obstruction
* 3
if)

O E ) Rupture of the chordae tendineae 0)


CL

Time
Correct Answer: E.
42. A 65-year-old man who has chronic lymphocytic leukemia has the sudden onset of fatigue and shortness of
breath. Laboratory studies show:
Hemoglobin
Hematocrit
6.5 g/dL
19%
•u to 0
I

Leukocyte count 50,000/mm 3 ridp


° o°#o %%
Platelet count 170,000/mm 3
al
Reticulocyte count 8%
Serum bilirubin
Total 4 mg/dL
Conjugated 0.5 mg/dL

A peripheral blood smear is shown. Which of the following is the most likely cause of the anemia? o c
O
®

0
A ) Aplastic anemia
B ) Autoimmune hemolysis
C ) Hereditary spherocytosis
D ) Microangiopathic hemolysis
o
*° o •
fJZ ^ o0
© 0
4

0 E ) Monoclonal gammopathy c
Correct Answer: B. t
,

y
43. A 20-month-old girl is brought to the physician by her mother because the mother is concerned about her daughter's development and behavior. The
girl is able to walk and run and has a vocabulary of at least 100 words. However, when she attends playdates with other children, she prefers to play
by herself and will strike another child on the head if that child attempts to join her in playing with a particular toy. Which of the following best
explains this patient's actions during play?

C A ) Attention-deficit/hyperactivity disorder
B ) Conduct disorder
0 C ) Early sign of autistic disorder
(5 D ) Oppositional defiant disorder

E ) Normal behavior for age


Correct Answer: E.
y
44. A 42-year-old man is struck by a motor vehicle. His only injury is a closed fracture of the proximal tibia. Initial neurovascular examination shows no
-
deficits. Twenty four hours later, he has increased leg pain and paresthesias in the dorsal space between his first and second toes. The patient
begins to pass dark red urine and becomes oliguric. Urinalysis is positive for blood but no erythrocytes are seen on microscopic examination. Which
of the following acute disorders is the most likely cause of the renal failure?

C A ) Glomerulonephritis
(• B ) Hemolytic-uremic syndrome
C ) Interstitial nephritis
D ) Nephrotic syndrome
O E ) Tubular necrosis
Correct Answer: E.

45. A 62-year-old man is being evaluated for rectal bleeding. An x-ray of the gastrointestinal tract with
contrast material is shown. Which of the following is the most likely explanation for the feathery
appearance in the portion of the gastrointestinal tract indicated by X when compared with the
portion indicated by Y?

0 A ) Absence of circular muscle


3 B ) Fewer villi
0 C ) Greater bowel motility
® D ) Greater mucosal surface area
3 E ) Less circular and longitudinal smooth muscle

Correct Answer: D.
y
46. An investigator conducts a phase 1 clinical trial to test the efficacy of a systemic antagonist to lymphocyte function-associated antigen 1 (LFA-1).
The drug is approved for topical administration to treat keratoconjunctivitis sicca in patients with multiple sclerosis. During the trial, it is found that
these patients are at an increased risk for bacterial infection as a result of this drug. The most likely explanation for this risk is drug-induced
blockade of which of the following segmented neutrophil activities?

( A ) Activation of phagocytosis
l
O B ) Adhesion to the endothelium
O C ) Diapedesis through the capillary wall
0 D ) Migration to the site of injury
0 E ) Rolling at the vessel periphery
Correct Answer: B.

/ 47. A 30-year-old man develops urinary incontinence 2 weeks after successful treatment of a fracture of the left pelvis that was sustained at work. He
was pinned against a loading dock by a truck moving in reverse. Physical examination shows a distended bladder. Cystometrography shows
absence of micturition reflexes. After the bladder fills to capacity, overflow of urine occurs through the urethra a few drops at a time. This patient
most likely sustained additional injury to which of the following during his initial accident?

O A ) External urinary sphincter


0 B ) Hypogastric nerve
® C ) Pelvic nerves
0 D ) Pudendal nerves
E ) Skeletal motor fibers
Correct Answer: C.

Normal AB Type 2 Diabetes Mellitus


Fasting Serum Glucose

48. During an investigational study, fasting serum glucose concentrations are collected from a population of healthy individuals and a population of
patients with type 2 diabetes mellitus. The results are shown in the graph. Switching from cut point A to cut point B would most likely result in which
of the following changes in test characteristics regarding identification of patients with type 2 diabetes mellitus?

( • A ) Decreased specificity
B ) Improved sensitivity
O C ) Increase in false-negatives
C D ) Increase in false-positives
3 E ) No change
Correct Answer: C.

y
49. A previously healthy 20-year-old woman comes to the emergency department because of a 3-day history of fever, shaking chills, headaches,
fatigue, and joint and muscle pain. She recently spent the summer working as a lifeguard on Long Island, New York. She has never traveled outside
the USA. She underwent splenectomy for injuries sustained in a motor vehicle collision at the age of 6 years. Her temperature is 39.1°C (102.4°F).
Physical examination shows no other abnormalities. A peripheral blood smear shows small intraerythrocytic rings; the result of a polymerase chain
reaction test for Plasmodium species is negative. Which of the following is the most likely causal organism?

O A ) Babesia microti
B ) Bartonella bacilliformis
C ) Bordetella bronchiseptica
• D ) Borrelia burgdorferi
E ) Brucella melitensis
Correct Answer: A.
y
50. Free purine and pyrimidine bases are reutilized in normal metabolism. In children with Lesch-Nyhan syndrome who have intellectual disability, poor
muscle coordination, and self-mutilation tendencies, there is a defect in the salvage of which of the following pairs of bases?

® A ) Adenine and thymine


O B ) Guanine and hypoxanthine
3 C ) Guanine and uric acid
0 D ) Uracil and cytosine
0 E ) Xanthine and hypoxanthine
Correct Answer: B.
PART 2
y
5. A 22-year-old woman who has P-thalassemia has a P-hemoglobin allele that carries a single base substitution (A — G) at the splice acceptor site of
the second intron. Which of the following events in expression of the P-hemoglobin gene is most likely to be impaired by this mutation?

C A ) Binding of nucleosomal histones to the transcript


(• B ) Cleavage of the transcript by RNA ligase
O C ) Modification of the transcript by small nuclear ribonucleoproteins
D ) Recognition of the transcript by signal recognition particle
C E ) Transport of the transcript out of the nucleus
Correct Answer: C.

y
6. A 28-year-old white woman comes to the physician for a follow-up examination. Two days ago, she was discharged from the hospital after being
treated for a hypertensive crisis with seizure. She has no other history of major medical illness. Current medications include a p-adrenergic blocker,
dihydropyridine calcium channel blocker, and a centrally acting agent stimulating presynaptic a 2 receptors. Her temperature is 37°C (98.6°F), pulse is
54/min, and blood pressure is 182/100 mm Hg. Physical examination shows trace ankle edema. Cardiac examination shows an S 4 gallop. A soft
abdominal bruit is heard left of the umbilicus. Abdominal ultrasonography shows normal-sized kidneys and a normal liver, spleen, pancreas, and
gallbladder; a small left ovarian cyst is noted. Laboratory studies show that complete blood count, serum electrolyte concentrations, and renal and
liver function test results are within the reference ranges. Which of the following is the most likely cause of this patient's secondary hypertension?

O A ) Chronic kidney disease


C B ) Coarctation of the aorta
O C ) Fibromuscular dysplasia of the left renal artery
(• D ) Hyperthyroidism
E ) Primary hyperaldosteronism
Correct Answer: C.

y
7. Receptors for all of the members of the steroid hormone family share which of the following features?


( ) A) Calcium-mediated intracellular effects
B ) GTP-binding proteins
C ) Mitochondrial membrane association
D ) Plasma membrane association
E ) Zinc-containing DNA-binding domains
Correct Answer: E.

y
8. A 52-year-old man who is in the hospital with malignant hypertension, heart failure, and a mild renal impairment is being treated with captopril,
nitroprusside, and furosemide. After 48 hours of therapy, his blood pressure is controlled, but he develops metabolic acidosis, lethargy, and difficulty
breathing. Which of the following substances most likely caused these adverse effects in this patient?

A ) Cyanide
B ) Iron
0 C ) Lead
(5 D ) Nitric oxide

0 E ) Selenium
Correct Answer: A.
9. A 10-year-old boy with spinal muscular atrophy is brought to the emergency department because of a 2-week history of nausea and vomiting. He
also has had decreased appetite during this period. One week prior to his illness, his entire family had similar symptoms which improved after a few
days. Physical examination shows decreased skin turgor, dry mucous membranes, and diffuse muscle weakness and atrophy. Laboratory studies
show:
Serum
Na + 140 mEq/L (N=138-145)
K+ -
4 mEq/L (N=3.4 4.7)
ci- 105 mEq/L (N=95-105)
HCO 3- 18 mEq/L (N=22-28)
Urea nitrogen 18 mg/dL (N=5-18)
Glucose 30 mg/dL (N=60-100)
Urine
pH 5.0 (N=4.6-8.0)
Specific gravity 1.020 (N=1.003-1.030)
Glucose none
Protein trace
Ketones 4+
RBC 0/hpf
WBC 0/hpf

Which of the following biochemical cycles in the liver is most likely providing this patient's brain with an energy source at this time?

O A ) Alanine-glucose cycle
® B ) Carnitine-acylcarnitine cycle
O C ) Lactic acid cycle
D ) Tricarboxylic acid cycle
O E ) Urea cycle
Correct Answer: B.

/ 10. An 18-month-old boy is brought to the emergency department because of lethargy for 3 hours. He has not eaten well for the past 24 hours. He had
cardiorespiratory arrest associated with hypoglycemia after an episode of diarrhea 10 months ago. On arrival, he is unresponsive. His temperature
is 37°C (98.6°F), pulse is 140/min, respirations are 25/min, and blood pressure is 100/60 mm Hg. Physical examination shows hepatomegaly.
Laboratory studies show:
Serum
Glucose decreased
Ketones decreased
Carnitine decreased
Urine
Ketones decreased
Dicarboxylic acids present

Following intravenous administration of glucose, he becomes responsive. Medium-chain triglycerides are administered daily for 6 months. At a
follow-up examination, his serum glucose concentration is within the reference range. A deficiency of which of the following enzymes is the most
likely cause of these findings?

A ) Glucose 6-phosphatase
O B ) HMG-CoA lyase
C ) Hormone-sensitive lipase
D ) Lipoprotein lipase
® E ) Long-chain acyl-CoA dehydrogenase
F ) Phosphoenolpyruvate carboxykinase
Correct Answer: E.
y

11. A 45-year-old man comes to the physician because of a 6-month history of progressive shortness of breath with exertion. He has a history of
frequent nosebleeds since adolescence. He is 178 cm (5 ft 10 in) tall and weighs 79 kg (175 lb); BMI is 25 kg/m 2. Physical examination shows the
findings in the photographs. Inhaled albuterol does not improve his symptoms. Which of the following is the most likely cause of this patient's
condition?

0 A ) Atrial septal defect


B ) Hypertension
O C ) Pulmonary arteriovenous shunting
2 D ) Reactive airway disease
® E ) Thromboembolism
Correct Answer: C.
y
12. To investigate the association between cellular telephone use and the development of brain tumors, 782 patients with brain tumors are compared
with 799 age- and sex-matched patients who do not have a malignancy. The results of the study find that there was no increase in the odds ratio for
cellular telephone use in patients who had brain cancer compared with those who did not have cancer. Which of the following best describes this
study design?

-
O A ) Case control study
@ B ) Case series
0 C ) Cross-sectional study
C D ) Prospective cohort study
0 E ) Randomized clinical trial
F ) Retrospective cohort study
Correct Answer: A.

y
13. A bronchial smooth muscle preparation is maintained in a tissue bath. Bioactive substances can be applied
to the smooth muscle, and contraction and relaxation of the smooth muscle can be measured. The results of
administration of thromboxane A 2 (TXA and prostaglandin E 2 (PGE 2) are shown. Which of the following is
^
the most likely mechanism of the effect of application of the PGE 2?

O A ) Activation of phospholipase A 2
® B ) Closing K + channels
O C ) Increase in cAMP
D ) Increase in inositol 1,4,5-trisphosphate
0 E ) Inhibition of 5-lipoxygenase
Correct Answer: C.

14. Histone acetyltransferases catalyze the acetylation of lysine residues in the amino-terminal tails of histones. Which of the following is the most
likely effect of this covalent modification on chromatin structure?

® A ) Decreases the affinity of histones for DNA


B ) Decreases the nucleosome content of the nucleus
C ) Increases the affinity of histones for DNA
D ) Increases the nucleosome content of the nucleus
E ) Removes histone H1 from DNA
F ) Removes histone H4 from DNA
Correct Answer: A.
y
15. A 65-year-old man who is a veteran comes to the office because of a 3-month history of increasingly severe pain of his low back. He has no history
of major medical illness. Physical examination shows point tenderness over the spine at the level of L5. Laboratory studies show:
Hemoglobin 9.8 g/dL
Hematocrit 28%
Serum
Ca 2+ 11.2 mg/dL
Creatinine 2.2 mg/dL
IgG 8000 mg/dL (N=650-1500)

X-rays of the spine show a compression fracture at L5 and four lytic lesions in the iliac crest. Examination of a bone marrow biopsy specimen
confirms the diagnosis and the presence of a chromosomal translocation. Treatment with a chemotherapeutic regimen, including a proteasome
inhibitor, is started. Which of the following is most likely to occur in the affected cells as a result of this therapy?

C A ) Decreased calcium release


B ) Decreased genomic instability
O C ) Decreased ubiquitination
® D ) Increased protein degradation
C E ) Induction of apoptosis
Correct Answer: E.

f
16. A 341-year-old man has a herniated lumbar intervertebral disc. Laminectomy and removal of the herniated fragment are scheduled. Which of the following labeled sites on the normal lumbar axial CT scan shown is the most appropriate
CX . ition for surgical entrance to the neural canal?

A)
• B)
C)
D)
E)
Correct Answer: D.

/ 17. A 15-year-old girl is brought to the physician by her mother because of a 1-year history of increasingly severe abdominal pain that occurs
periodically, but not regularly. Physical examination shows abdominal tenderness that is most acute around the umbilicus. The physician suspects
ectopic endometrial tissue, and an operation is scheduled. During the operation, an 8-cm piece of tissue is removed from the ileal region on the
antimesenteric side of the bowel approximately 90 cm proximal to the ileocecal junction. The patient’s pain resolves postoperatively. The ectopic
tissue was most likely located in which of the following structures?

O A ) Ascending colon
O B ) Falciform ligament
O C ) Ileal polyp
® D) Meckel diverticulum
O E ) Vermiform appendix
Correct Answer: D.

18. A 35-year-old woman and her 35-year-old husband come to the physician for genetic counseling after their son is diagnosed with a rare metabolic
disease. Assuming Hardy-Weinberg equilibrium, the physician tells the parents about the incidence and carrier frequency of this disorder. Which of
the following is most likely to disturb the Hardy-Weinberg equilibrium of this disorder?

® A ) Appreciable rate of gene mutation


3 B ) Random matings in the population
C ) Relatively large population
D ) No selection against a certain genotype
E ) No significant immigrant population
Correct Answer: A.
y
19. An 1814-g (4-lb) female newborn is delivered in the hospital at 24 weeks' gestation to a 25-year-old primigravid woman via a spontaneous vaginal
delivery. Examination of the newborn shows no peripheral cyanosis. There is a grade 3/6, continuous murmur over the left midclavicular, second
intercostal space. At the age of 2 days, the newborn develops respiratory distress. Echocardiography shows a failure of a vascular structure to
close. The embryologic origin of this anomaly is most likely which of the following aortic arches?

A ) First
B ) Second
C ) Fourth
(•> D ) Fifth
E ) Sixth
Correct Answer: E.

20. A 45-year-old man comes to the physician because of progressively worsening, constant pain in
his left thigh over the past 3 months. He is a long-distance runner. An x-ray of the femur shows
thickening of the diaphysis and disruption of the cortex with focal areas of increased calcification. A
glistening mass containing several cysts is surgically excised. A photomicrograph of tissue from the
mass is shown. Which of the following is the most likely diagnosis?

O A ) Atypical stress fracture


® B ) Chondrosarcoma
0 C ) Enchondroma
O D ) Ewing sarcoma
3 E ) Giant cell carcinoma
O F ) Multilobulated bone cyst
O G ) Osteosarcoma

*- ,1
Correct Answer: B.
r?
-
y
21. An 81-year-old woman is admitted to the hospital because of a massive pulmonary embolism from a deep venous thrombosis. Her platelet count is
160,000/mm 3. Appropriate pharmacotherapy is started. One week later, her platelet count is 55,000/mm 3. The thrombocytopenia was most likely
caused by a drug with which of the following mechanisms of action?

C A ) Activates tissue plasminogen


B ) Interferes with the carboxylation of coagulation factors
0 C ) Irreversibly inactivates cyclooxygenase
D ) Potentiates the action of antithrombin III
E ) Selectively inhibits factor Xa
Correct Answer: D.
* 2 2. A 56-year-old man dies 1 month after the onset of difficulty with short-term memory and anxiety. He underwent surgical excision of small cell
carcinoma 1 year ago. Degenerative changes are most likely to be concentrated at which of the following labeled sites on the normal MRI of the
head?

3 A)
B)
@ C)
D)
E)
F)
Correct Answer: C.

y
23. A 68-year-old woman comes to the physician because of a 1-year history of severe abdominal pain after meals; she also has had an unintentional
9-kg (20-lb) weight loss during this period. The pain is relieved when she decreases the amount of food that she eats. She has a history of
atherosclerosis and underwent triple coronary artery bypass grafting 2 years ago. Physical examination shows a soft, nontender abdomen and an
abdominal bruit. Pedal pulses are diminished. The most likely cause of this patient's symptoms is stenosis of which of the following arteries?

G A ) Greater pancreatic
G B ) Hepatic
C ) Right gastric
D ) Superior mesenteric
E ) Supraduodenal
Correct Answer: D.

y
24. An investigator obtains a serum sample from an individual previously immunized with a vaccine that elicits immunity to Clostridium tetani.
Administration of this serum to a guinea pig prior to challenge with a potentially lethal dose of C. tetani is most likely to mediate protection to the
challenge dose through binding of antibody to which of the following products of the bacterial cells?

G A ) Capsular polysaccharide
B ) Cell wall polysaccharide
C ) Exotoxin
D ) Fimbria
E ) Flagellum
F ) Peptidoglycan
Correct Answer: C.
y

25. An 18-year-old man is brought to the emergency department 30 minutes after sustaining injuries in a motor vehicle collision. He has abdominal pain. His pulse is 120/min, and
blood pressure is 80/60 mm Hg. Physical examination shows multiple contusions over the trunk and abdominal tenderness. A chest x-ray shows multiple rib fractures. A CT
scan of the abdomen is shown. Which of the following organs is most likely injured in this patient?

O A ) Bladder
® B ) Liver
C ) Pancreas
O D ) Spleen
O E ) Stomach
Correct Answer: D .

y
26. A 15-year-old girl is brought to the physician because of a 3-day history of fever, sore throat, and malaise. Her temperature is 39.2°C (102.6°F). Physical examination shows
diffuse pharyngeal erythema, moderately enlarged tonsils, and tender anterior and posterior cervical lymphadenopathy. A complete blood count shows:
Leukocyte count 19,500/mm3 (N=3500-10,500)
Segmented neutrophils 30%
Bands 7%
Eosinophils 2%
Lymphocytes 25%
Lymphocytes, atypical 30%
Monocytes 6%

Incubation of this patient’s serum with sheep erythrocytes results in agglutination. The atypical lymphocytes in this patient are most likely which of the following cell types?

® A ) B lymphocytes
O B ) CD4+ T lymphocytes
O C ) CD8+ T lymphocytes
D ) FOXP3-expressing regulatory T lymphocytes
O E ) Natural killer cells
Correct Answer: C.
y
27. A 68-year-old man comes to the physician's office because of the sudden onset of loss of vision in the left eye. There is no pain. He has a 15-year
history of well controlled type 2 diabetes mellitus and hypertension. He had a myocardial infarction 2 years ago. Temperature is 36.7°C (98.1°F),
pulse is 72/min, respirations are 16/min, and blood pressure is 200/100 mm Hg. Funduscopic examination shows a pale, white retina with retinal
artery narrowing and decreased filling of the retinal veins. Which of the following is the most likely cause of the loss of vision?

C A ) Malignant hypertension
O B ) Retinal artery embolism
• C ) Retinal detachment
O D ) Retinal vein thrombosis
O E ) Vitreous hemorrhage secondary to neovascularization
Correct Answer: B.

/ 28. A 55-year-old woman comes to the physician because of a 3-year history of weakness, fatigue, decreased appetite, and constipation. She says
that she did not receive medical treatment for these symptoms because she did not have health insurance until recently. She was admitted to the
hospital 1 year ago because of renal calculi, but she could not afford to pay for follow-up care. X -rays at that time were indicative of osteitis fibrosa
cystica. Vital signs are within normal limits. Physical examination shows bony tenderness and muscle strength of 4/5 with decreased tone. Serum
studies show an increased calcium concentration and a decreased phosphorus concentration. Which of the following is the most likely diagnosis?

A ) Adult T-lymphocyte leukemia


® B ) Chief cell adenoma of the parathyroid gland
C ) Familial (benign) hypocalciuric hypercalcemia
D ) Medullary carcinoma of the thyroid gland
E ) Multiple myeloma
Correct Answer: B.
y
29. A 52-year-old man with chronic renal failure receives a cadaveric kidney transplant. Postoperatively,
he is given cyclosporine for immunosuppressive therapy. Six weeks after the operation, he develops
hypoxemia with an arterial Po 2 of 40 mm Hg. A photomicrograph of a transbronchial biopsy
specimen is shown. Which of the following pathophysiologic mechanisms best explains this patient's
hypoxemia?

0 A ) Alveolar exudation due to Cryptococcus neoformans infection


B ) Cytomegalovirus pneumonitis with diffuse alveolar damage
® C ) Neutrophilic alveolar consolidation due to pneumococcal pneumonia
D ) Pneumocystis jirovecii (formerly P. carinii ) infection with alveolar exudation
E ) Vascular invasion by Aspergillus with pulmonary infarction

Correct Answer: B.

y
-
30. A 58-year old woman comes to the office for a follow-up examination 2 days after she noticed a lump in her left breast on self-examination.
Menopause occurred 6 years ago. Physical examination shows a 1-cm, poorly circumscribed, nodular mass in the left periareolar tissue.
Mammography shows an ill-defined density suspicious for malignancy. A needle biopsy specimen is nondiagnostic. Microscopic examination of the
excised mass shows a large dilated space lined by glandular epithelium and surrounded by a dense lymphoplasmacytic inflammatory infiltrate; the
associated lumen is filled with lipid-laden macrophages and amorphous debris. Which of the following is the most likely diagnosis?

0 A ) Acute mastitis
3 B ) Fibroadenoma
® C ) Fibrocystic changes
D ) Intraductal carcinoma
O E ) Mammary duct ectasia
Correct Answer: E.

y
31. A 3-year-old girl is brought to the physician because of a 2-week history of easy bruisability. She has had frequent, large, foul-smelling stools for
3 months. She was found to have cystic fibrosis at the age of 18 months. She is below the 3rd percentile for height and weight. Physical
examination shows several ecchymotic areas over the extremities. This patient most likely has a deficiency of which of the following?

® A ) Antithrombin III
3 B ) Platelets
3 C ) Protein C
D ) Vitamin B 12 (cobalamin)
3 E ) Vitamin C
3 F ) Vitamin K
3 G ) von Willebrand factor
Correct Answer: F.

/ 32. A 75-year-old man with alcoholism is brought to the emergency department 30 minutes after he began vomiting blood. He has a history of portal
hypertension. Abdominal examination shows spider angiomata. Which of the following is the most likely cause of this patient's hematemesis?

3 A ) Anastomosis between the superior and inferior mesenteric veins


B ) Aortoesophageal fistula
C ) Inflammation of the distal esophagus
® D ) Retrograde blood flow between tributaries of the left gastric to the azygos veins
E ) Shunting of blood from the portal paraumbilical veins to the superior epigastric veins
Correct Answer: D.
y
33. A study is conducted to assess 32 patients in a community of 1000 who have developed drug-resistant tuberculosis during a 1-year period. These
patients are removed from the community for treatment. Assuming that the risk for infection and susceptibility to the disease is constant, which of
the following best represents the number of individuals most likely to develop subsequent drug-resistant tuberculosis during the next year?

O A ) 27
8 B ) 29
C ) 31
3 D ) 32
3 E ) 33
Correct Answer: C.

y
34. A 63-year-old woman comes to the physician because of a 5-day history of shortness of breath and swollen legs. Her respirations are 25/min and
labored, and blood pressure is 130/50 mm Hg. She has a large subclavian arteriovenous fistula caused by a stab wound to the left supraclavicular
area 15 years ago. Physical examination shows 2+ edema of the lower extremities. Which of the following findings is most likely in this patient?

3 A ) Decreased arterial oxygen saturation


B ) Decreased mixed venous oxygen saturation
3 C ) Decreased stroke volume
D ) Increased resting cardiac output
($ E ) Increased systemic vascular resistance

Correct Answer: D.

y
35. A 78-year-old man comes to the physician with his daughter because of low back pain for 3 months. He underwent a bilateral orchiectomy 2 years
ago for prostate cancer. He speaks French fluently, but he knows only a few English words. His daughter offers to serve as an interpreter. After
thanking the daughter for her offer, it is most appropriate for the physician to state which of the following?

(5 A ) "Have you interpreted for your father before in a medical capacity?"

B ) "I believe it would be awkward for a daughter to interpret for her father."
O C ) "It is best that I use a professional French interpreter."
3 D ) "Please ask your father if he's ready to begin the examination."
3 E ) "That would be fine. I'll just ask you to step out of the room for the physical examination."
Correct Answer: C.

y
-
36. A 22 year-old man is admitted to the hospital for treatment of a ruptured appendix. His temperature is 38.9°C (102°F). Abdominal examination
shows guarding, rebound tenderness, and extreme tenderness of the right lower quadrant. He undergoes exploratory laparotomy, and an abscess
containing foul-smelling pus is found. A Gram stain of exudate shows numerous segmented neutrophils, gram-positive cocci, gram-positive rods,
and gram-negative rods. Aerobic culture of the exudate at 37°C on blood and MacConkey agar plates yields only enterococci. Which of the following
bacteria is most likely to be among the gram-negative rods?

O A ) Bacteroides fragilis
3 B ) Escherichia coli
C ) Legionella pneumophila
• D ) Moraxella catarrhalis
E ) Proteus mirabilis
Correct Answer: A.

y
37. A newborn delivered at 28 weeks' gestation is in severe respiratory distress. The immature alveoli of this newborn's lungs have a diminished ability
to serve as sites of effective gas exchange. An increase in which of the following parameters best explains this finding?

3 A ) Alveolar radii
@ B ) Lung compliance
3 C ) Lung elastic recoil
D ) Pleural pressure
E ) Surfactant secretion
Correct Answer: C.
38. A 7-year-old boy is about to undergo an appendectomy. An intravenous catheter needs to be inserted, but the patient is extremely fearful of being
stuck with a needle while awake. The most appropriate anesthesia administered by mask to anesthetize this patient quickly would have which of
the following characteristics?

O A ) High blood solubility


B ) High cerebrospinal fluid solubility
3 C ) High lipid solubility
® D ) Low blood solubility
E ) Low lipid solubility
Correct Answer: D.

y
39. A 76-year-old woman comes to the physician because of a 2-day history of abdominal pain, nausea, and vomiting. Her temperature is 38.9°C
(102°F), pulse is 88/min, respirations are 26/min, and blood pressure is 117/79 mm Hg. Abdominal examination shows tenderness of the left lower
quadrant. Laboratory studies show a leukocyte count of 22,500/mm 3 with 8% bands. A CT scan of the abdomen shows an abscess in the left lower
quadrant. The patient undergoes drainage of the abscess and a partial sigmoidectomy for a ruptured diverticulum. Neutrophil activation in this
patient is directed, in part, by bacterial lipopolysaccharides. As a result, these lipopolysaccharides most likely bind to which of the following
neutrophil receptor types?

0 A ) Cytokine
(S B ) G protein-linked

0 C ) Ion channel-linked
D ) Nuclear
E ) Toll-like
Correct Answer: E.

y
40. A 2-year-old boy is brought to the physician because of a 6-month history of failure to thrive. Cardiac examination shows a grade 4/6 systolic
murmur caused by increased pulmonic flow, followed by a fixed, widely split Sj. Echocardiography shows hypertrophy of the right atrium, right
ventricle, and pulmonary arteries. This patient most likely has which of the following congenital cardiac anomalies?

O A ) Atrial septal defect


B ) Patent ductus arteriosus
C ) Persistent truncus arteriosus
D ) Tetralogy of Fallot
• E ) Transposition of the great arteries
F ) Ventricular septal defect
Correct Answer: A.

y
41. A homeless 45-year-old man comes to the clinic because of localized back pain for the past 3 months. He has not seen a health care provider in
more than 20 years. Physical examination shows tenderness over the lower thoracic spine. There are no obvious neurologic deficits. X-rays of the
spine show narrowing of the disc space between T10 and T11 and destruction of the adjacent T10 vertebral body. Which of the following is the most
likely cause of these findings?

(
i A ) Hyperparathyroidism
B ) Multiple myeloma
0 C ) Osteitis deformans (Paget disease)
D ) Sarcoidosis
O E ) Tuberculosis
Correct Answer: E.

y
42. A 25-year-old man comes to the physician because of sores on his penis for 2 days. Physical examination shows vesicular and ulcerative lesions on
the penis. Drug therapy is initiated. One week later, his symptoms resolve. Which of the following enzymes is required to activate the most
commonly prescribed drug?

O A ) ATPase
B ) Cytochrome P450
0 C ) Protease
3 D ) Reverse transcriptase
O E ) Thymidine kinase
Correct Answer: E.
This patient’s vesicular and ulcerative lesions on the penis are likely caused by herpes simplex virus in the form of genital herpes. Treatment for herpetic
y
43. A 40-year-old man has moderate edema. A 24-hour urine collection contains 15 g of protein (N£150 mg/24 h). Which of the following is the most
likely mechanism of the edema?

0 A ) Decreased capillary hydrostatic pressure


O B ) Decreased plasma oncotic pressure
0 C ) Increased plasma oncotic pressure
D ) Decreased permeability of postcapillary venules
(• E ) Increased permeability of postcapillary venules
Correct Answer: B.

44. A 14-year-old girl is brought to the physician by her mother because of episodes of increasingly severe facial blemishes during the past 6 months.
-
She says that she does not eat high fat foods or use makeup. Use of an astringent soap has not resolved her symptoms. Menarche occurred at
the age of 12 years. A photograph of her face is shown. Which of the following best describes the pathologic mechanism of this patient's condition?

O A ) Acceleration of the proliferation of cells in the dermis by increased estrogen production


B ) Colonization of apocrine sweat glands by fungi
D C ) Decreased blood flow to skin capillaries by overproduction of elastic fibers
® D ) Follicular epidermal hyperproliferation with excess production of sebum
E ) Thickening of the basement membrane in response to increased pubertal serum estrogen concentrations
Correct Answer: D.

Cell Membrane Phospholipids


45. The illustration shows the arachidonic cascade that is activated when skin is exposed to poison ivy. Within
this cascade, which of the following is the major mechanism of analogs of corticosteroids?

A ) Acceleration of the metabolism of arachidonic acid


B ) Inhibition of receptors mediating selected responses to prostaglandins and leukotrienes
Cyclooxygenase ^
J[
Phospholipase A2
Arachidonic Acid
Lipoxygenase

C ) Nonselective inhibition of cyclooxygenase and lipoxygenase Prostaglandins Leukotrienes

® D ) Regulation of phospholipase A 2 II
Tissue receptors
II
Tissue receptors
E ) Selective inhibition of lipoxygenase
Correct Answer: D.

y
46. A 32-year-old woman comes to the physician because of a 3-month history of headaches. Two weeks ago, she moved to an area women's shelter
because her husband had been hitting her. She takes no medications. Physical examination shows multiple bruises in different stages of healing
over the face, neck, and trunk. There is circumferential bruising on both sides of the neck. Which of the following statements is most appropriate to
gather pertinent information from this patient?

C A ) "Did you bring your records from the emergency room with you?"
O B ) "I see some bruising on your neck. How did that happen?"

( ) C ) "Please demonstrate to me how this neck bruising occurred."

0 D ) "This looks like you were choked. Is that true?"


E ) "What sort of device did your husband use on your neck?"
Correct Answer: B.
PART 3
y
1. A 43-year-old woman comes to the physician because of a 6-month history of altered consciousness. During the interview, she stops talking in mid-
sentence, turns her head to the right, and extends and stiffens her right upper extremity. She has a blank look and does not respond to any
questions. She then has some repetitive lip smacking and picking movements of the hands. The episode lasts approximately 30 seconds. She slowly
returns to her normal state during the next 4 to 5 minutes. The most likely diagnosis is which of the following types of seizures?

O A ) Absence
O B ) Complex partial
0 C ) Generalized tonic-clonic
(£ D ) Simple partial motor

E ) Simple partial sensory


Correct Answer: B.

- -
2. A 22 year-old woman of Irish descent is brought to the emergency department because of a 1 hour history of severe pain and coldness of her left
leg. She has an IQ of 80, and she dropped out of high school in the tenth grade. Her parents and two older siblings have normal intelligence and
have no history of these symptoms. Ophthalmologic examination shows a partially dislocated lens in the right eye. Physical examination shows
mottling and loss of pulses in the left lower extremity and foot. Arteriography of the left lower extremity shows thrombosis of the femoral artery. This
patient most likely has a metabolic disorder involving which of the following amino acids?

A ) Glycine
0 B ) Homocyst(e)ine
C ) Leucine
D ) Phenylalanine
E ) Tyrosine
Correct Answer: B.

y
3. An investigator attempting to develop a vaccine for gonorrhea produces an attenuated organism that he administers to female rats as an inhaled
aerosol. Swabs of vaginal secretions show a significant increase in antibodies specific for gonorrheal antigens. Which of the following mechanisms
best accounts for this immune response in the genital tract?

C A ) Fecal-oral transfer of attenuated organisms


B ) Synthesis of IgG by circulating lymphocytes exposed to antigens in the lung
O C ) Trafficking of IgA-producing lymphocytes to seed all mucosal sites
• D ) Transport of attenuated Neisseria gonorrhoeae from the lung to the vaginal mucosa
E ) Transport of circulating IgM across the vaginal epithelium
Correct Answer: C.

y
4. A 50-year-old woman from Egypt comes to the physician because of painless blood in her urine for 2 weeks. Urinalysis is within the reference range.
Urine cytologic findings show malignant squamous epithelial cells. Which of the following is the strongest predisposing risk factor for this type of
tumor?

0 A ) Chronic pyelonephritis
0 B ) Cigarette smoking
C ) Diabetic nephropathy
3 D ) Excessive consumption of caffeine
0 E ) Long-term use of phenacetin
F ) Schistosoma haematobium infection
Correct Answer: F.

y
5. The higher incidence of birth defects in children born to women more than 35 years old may be the result of environmental influences on
chromosomes in developmentally arrested oocytes at which of the following stages of cell division?

A ) Prophase, meiosis I
B ) Prophase, meiosis II
C ) Metaphase, meiosis I
0 D ) Metaphase, meiosis II
E ) Telophase, meiosis I
Correct Answer: A.
/

6. A 65-year-old man with diabetes mellitus develops a neuropathy of the cranial nerve indicated by the arrow in the
normal brain stem. Which of the following is the most likely associated functional deficit?

A ) Complete ptosis of the ipsilateral eye


(• B ) Inability to abduct the ipsilateral eye
0 C ) Ipsilateral sensorineural deafness
D ) Lack of tear formation in the ipsilateral eye
E ) Paralysis of ipsilateral jaw muscles

| Correct |

Correct Answer: B.

7. A 42-year-old man comes to the physician because of a 1-week history of a diffuse, painless rash on his back. He first noticed the rash after he spent
a week sunbathing at the beach. He says the rash does not itch. A photograph of the rash is shown. Which of the following is the most likely causal
organism?

O A ) Malassezia furfur
B ) Staphylococcus aureus
0 C ) Streptococcus pyogenes (group A)
D ) Treponema pallidum
(5 E ) Trichophyton rubrum

Correct Answer: A.

y
8. A 6-year-old boy is brought to the emergency department because of coughing, wheezing, and rapid breathing for 6 hours. He was diagnosed with an
upper respiratory tract infection 2 days ago. His temperature is 37.2°C (99°F), pulse is 120/min, respirations are 44/min, and blood pressure is 90/60
mm Hg. Inspiratory and expiratory wheezes are heard throughout the lung fields. There is decreased tactile fremitus. Which of the following is the
most likely diagnosis?

A ) Asthma
B ) Atelectasis
0 C ) Bronchitis
(s D ) Left-sided heart failure
E ) Pneumococcal pneumonia
Correct Answer: A.
y
9. A 50-year-old man comes to the physician because of a 2-week history of progressive shortness of breath while climbing stairs to his office every
morning. He reports no other problems, but he is concerned because his father had a major myocardial infarction at the age of 52 years. His pulse is
110/min and regular, respirations are 16/min, and blood pressure is 135/95 mm Hg. The lungs are clear to auscultation. Cardiac examination shows
normal heart sounds with a physiologic split of S? Stress echocardiography shows hypokinesis of the posterior left ventricle with increasing activity
levels. Which of the following is the most likely cause of the posterior left ventricular findings in this patient?

C A ) Disruption of the sympathetic nerves to the left ventricle



( B ) Extravascular compression of the coronary arteries
0 C ) Increased left ventricular end-diastolic pressure
D ) Increased myocardial oxygen consumption
O E ) Stenosis of the right coronary artery
Correct Answer: E.

y
10. A 68-year-old man comes to the physician because of a 1-month history of not being able to sustain an erection for sexual intercourse, although he
has no difficulty during masturbation. His wife of 40 years died 2 years ago after a prolonged illness. He began dating recently. He has a
multinodular goiter but takes no medications. His serum thyroid-stimulating hormone concentration is 4.0 pU/mL, and serum testosterone
concentration is within the reference range. He does not have any symptoms of major depressive disorder. Which of the following pairs of additional
findings is most likely to be found on history taking?

Libido Nocturnal Erections


® A) Decreased decreased
3 B) Decreased normal
0 C) Normal decreased
3 D) Normal normal
Correct Answer: D.

y
11. A 46-year-old woman comes to the office because of a 3-month history of fever, malaise, and easy bruising. Her mother and maternal grandmother
died of breast cancer at the ages of 49 and 52 years, respectively. Her temperature is 38°C (100.4°F), pulse is 115/min, respirations are 16/min, and
blood pressure is 125/90 mm Hg. Physical examination shows numerous large ecchymoses over the trunk and lower extremities. Laboratory studies
show an increased leukocyte count with immature segmented neutrophils. Genetic testing shows the presence of the Philadelphia (Ph 1)
chromosome. Which of the following best explains this patient's current condition?

A ) ABL activation of the p23 tumor suppressor gene


0 B ) Fusion protein influence on the cell cycle
C ) Point mutation in the ABL oncogene
D ) Point mutation in the BCR gene
E ) Underexpression of the p53 tumor suppressor gene
Correct Answer: B.

y
12. An 11-year-old boy receives succinylcholine and sevoflurane for anesthesia during surgical repair of a hernia. During the procedure, the patient
develops muscle rigidity, tachycardia, hyperkalemia, hypocalcemia, and hyperthermia. This patient most likely carries a variant form of the gene
coding for which of the following?

C A ) A/-Acetyltransferase

( B ) Cytochrome P450 2D6
0 C ) Cytochrome P450 3A4
D ) Glutathione synthetase
O E ) Ryanodine receptor
Correct Answer: E.

y
13. While digging in the forest, an archaeologist finds the skeleton of a child in a shallow grave. Examination of the skull shows frontal bossing; there is
delayed eruption of the teeth and abnormal enamel formation of those teeth that are erupted. The spine and arm bones show no abnormalities. The
tibias show marked bowing. Which of the following was the most likely cause of this child's skeletal defects?

A ) Osteitis deformans (Paget disease)


B ) Osteogenesis imperfecta
0 C ) Rheumatic fever

( D ) Tuberculosis
,
E ) Vitamin B (thiamine) deficiency
O F ) Vitamin D deficiency
Correct Answer: F.
y

2
B
£

Time

14. The diagram depicts a cardiac murmur heard in a healthy 92-year-old man. Which of the following age-related processes is responsible for the
murmur?

0 A ) Calcification of the mitral valve


G B ) Enlargement of the mitral valve annulus
C ) Fibrosis and calcification of the aortic valve
D ) Myxomatous degeneration of the aortic valve cusp
E ) Myxomatous degeneration of the mitral valve
Correct Answer: C.

y
15. A 2-month-old boy is brought to the physician because of a 7-day history of hoarseness. His mother has AIDS and used cocaine throughout her
pregnancy. Physical examination shows no abnormalities. A fiberoptic laryngoscopic examination shows a nodule on the left vocal cord. Which of
the following viruses is the most likely cause of this nodule?

(• A ) Cytomegalovirus
O B ) Herpes simplex virus
C ) HIV
O D ) Human papillomavirus
O E ) Varicella-zoster virus
Correct Answer: D.

y
16. In biosynthesis of thyroid hormones, iodide is concentrated by the thyroid gland and oxidized by thyroidal peroxidase to iodine. Iodine iodinates
amino acid residues within the thyroglobulin molecule. Which of the following amino acids undergoes iodination?

0 A ) Histidine
B ) Phenylalanine
0 C ) Proline

( D ) Tryptophan
O E ) Tyrosine
Correct Answer: E.

y
17. A healthy 70-year-old woman has participated in a longitudinal study of the effects of aging on performance during pulmonary function tests for the
past 50 years. She has undergone a complete set of tests, including arterial blood gas analyses, every 5 years. Which of the following sets of
changes best represents this woman now compared with her results at the age of 20 years?

-
Alveolar arterial Po 2
Residual Volume Arterial Po 2 Difference
O A) t T I
B) t I t
• C) No change t T
3 D) No change T 1
3 E) 1 t 1
3 F) I i 1
Correct Answer: B.
y
18. A 64-year-old man with a 25-year history of alcoholism is brought to the emergency department by his wife because of a 1-day history of confusion.
He is disoriented and disheveled. His pulse is 110/min, respirations are 20/min, and blood pressure is 100/64 mm Hg. Physical examination shows
signs of dehydration and jaundice, and spider angiomata over the face and chest. There is a flapping, up-and-down motion of the hands when the
upper extremities are outstretched horizontally. Abdominal examination shows distention with bulging flanks and shifting dullness. In addition to
other appropriate pharmacotherapy, administration of oral neomycin is begun. Which of the following primary mechanisms of action is most likely to
occur in this patient as a result of this drug treatment?

A ) Binding of ammonia and other hepatically cleared toxins in the gut


(5 B ) Blockade of new protein synthesis by the liver

C ) Blockade of two successive steps in the metabolism of folic acid


0 D ) Increased endotoxin production where bacterial overgrowth occurs in the gut
C E ) Killing of bacteria in the gut that generate ammonia
Correct Answer: E.

19. An investigational drug (Drug X ) under development for the treatment of hypertension is being tested in phase 1 clinical trials. A significant number
of subjects develop dose-dependent increases in serum AST and ALT activities. The investigators hypothesize that this effect is due to oxidative
stress. Assuming that biopsy samples of the liver can be obtained from the subjects, which of the following would be the best measure of hepatic
oxidative stress caused by Drug X?

0 A ) Concentration of mRNAfor C-reactive protein


3 B ) Detection of nitrated proteins in hepatic cell membranes
0 C ) Microsomal cytochrome P450 concentration
D ) Rate of state 3 mitochondrial respiration
® E ) Ratio of GSH:GSSG
Correct Answer: E.

/ 20. A 28-year-old woman comes to the physician because of a 4-day history of palpitations, severe neck pain, fatigue, and malaise. Her pulse is
120/min and regular. Physical examination shows a diffusely tender, mildly enlarged thyroid gland. There is no exophthalmos. Serum studies show
a thyroid-stimulating hormone concentration of 0.01 pU/mL. Which of the following is the most likely diagnosis?

O A ) Factitious thyrotoxicosis
B ) Graves disease
® C ) Subacute granulomatous thyroiditis
O D ) Thyroid abscess
O E ) Toxic multinodular goiter
Correct Answer: C.

y
21. A 56-year-old man with esophageal varices who is awaiting liver transplantation undergoes placement of a transjugular portosystemic shunt. During
the procedure, a catheter with a stent is threaded through the right internal jugular vein and down through the vena cava to the liver. To relieve the
portal hypertension via this route, the stent should be placed between a branch of the portal vein and which of the following additional veins?

® A ) Esophageal
O B ) Hepatic
O C ) Left gastric
3 D ) Splenic
3 E ) Superior mesenteric
Correct Answer: B.
y
22. A 6-year-old girl is brought to the physician for a follow-up examination. She has been receiving treatment with potassium citrate monohydrate since
the diagnosis of renal tubular acidosis was made at the age of 2 years. Physical examination shows no abnormalities. Serum studies show:
Na + 142 mEq/L
K+ 3.5 mEq/L
Cl- 115 mEq/L
HCO 3- 18 mEq/L
Urea nitrogen 9 mg/dL
Creatinine 0.9 mg/dL

A defect in renal ammoniagenesis is suspected. Which of the following substrates is the most likely source of ammonia production in this patient?

C A ) Creatinine
B ) Glutamine
® C ) Glycine
0 D ) Leucine
0 E ) Urea
Correct Answer: B.

y
23. An 8-year-old girl is brought to the physician by her parents because of abdominal pain for 3 days. She is at the 90th percentile for height and the
80th percentile for weight. Breast, pubic hair, and axillary hair development is Tanner stage 3. Pelvic examination shows a firm, smooth, right
ovarian mass. Ultrasonography of the abdomen confirms the presence of an 8-cm-diameter ovarian mass. Which of the following types of ovarian
lesions is most likely in this patient?

C A ) Cystadenocarcinoma
B ) Dermoid cyst
C ) Embryonal rhabdomyosarcoma
D ) Fibroma
O E ) Granulosa cell tumor

( ) F ) Krukenberg tumor
G ) Papillary carcinoma
Correct Answer: E.

Y 24. An 18-month-old girl is brought to the physician because of failure to thrive and frequent wet diapers. She is at the 3rd percentile for length and
weight. Physical examination shows dehydration, decreased muscle tone, and bowing of the legs. Slit lamp examination shows crystals in the
corneas. Serum studies show hypophosphatemia, hypokalemia, and a nonanion gap metabolic acidosis. Urine studies show a pH of 5, glucosuria,
phosphaturia, and generalized aminoaciduria. Which of the following is the most likely cause of the metabolic acidosis in this patient?

® A ) Decreased carbonic anhydrase activity in the distal tubule


B ) Decreased carbonic anhydrase activity in the proximal tubule
C ) Decreased hydrogen ion secretion in the distal tubule
D ) Decreased hydrogen ion secretion in the proximal tubule
E ) Decreased sodium bicarbonate reabsorption in the distal tubule
O F ) Decreased sodium bicarbonate reabsorption in the proximal tubule
Correct Answer: F.

y
25. A 45-year-old woman with chronic myelogenous leukemia comes to the physician for a follow-up examination. Physical examination shows pallor
and bleeding gums. Laboratory studies show a leukocyte count of 90,000/mm 3 (60% blasts) and a platelet count of 27,000/mm 3. A drug regimen of
mitoxantrone, vinblastine, and thioguanine is initiated, with no improvement. During the last cycle of scheduled treatment, a sample of the blast cells
is harvested, and the intracellular concentrations of mitoxantrone and vinblastine in these blast cells are found to be undetectable. An increase in
which of the following mechanisms best explains this observation?

O A ) Drug efflux pump P-glycoprotein activity in leukemic cells


B ) Drug metabolism by the hepatic CYP450 system
® C ) Drug metabolism by novel heme enzymes in leukemic blast cells
D ) Production of sphingomyelin by the leukemic cells
3 E ) Renal excretion of the drugs
F ) Thiopurine methyltransferase activity in the leukemic cells
Correct Answer: A.
y
26. A 47-year-old woman comes to the physician because she wants to "do something about my wrinkles." Physical examination shows fine wrinkles in
photodamaged skin on the face. Treatment with tretinoin is started. Collagen synthesis will most likely be increased in this patient by which of the
following mechanisms?

A ) Activation of nuclear gene transcription


B ) Decreased cAMP production
0 C ) Displaced vitamin A from cellular stores
(t D ) Increased sebum production
E ) Protection of keratinocytes from UVB irradiation
Correct Answer: A.

y
27. A 60-year-old woman is receiving cisplatin therapy for advanced transitional cell bladder cancer. She develops
paresthesias, and the medication is stopped. The medication likely damaged the largest cells in the region labeled
"B" in the drawing of the spinal cord shown. The damage to these cells would most likely lead to which of the
following motor signs?


( A ) Babinski sign
Z B ) Clonus
0 C ) Fibrillations
O D ) Hyporeflexia
0 E ) Muscle atrophy
Correct Answer: D.

28. A 42-year-old man has an autoimmune disorder resulting in proximal muscle weakness of the lower extremities. Arrows in the photomicrograph
shown indicate membranes that contain high concentrations of channels that are targeted by this condition. Which of the following is the most likely
diagnosis?

0 A ) Multiple sclerosis
B ) Myasthenia gravis
C ) Myasthenic (Lambert-Eaton) syndrome
® D ) Periodic paralysis
y
29. An investigator is studying Helicobacter pylori strains isolated from the same patient over several years. The immune response to proteins produced
by these strains is observed. It is found that the original H. pylori isolate from the patient expresses one protein recognized by the patient's
antibodies, but subsequent isolates do not express this protein. Sequencing of the gene encoding the protein from the original and subsequent
isolates is done. It is found that subsequent isolates have nine consecutive cytidine residues, whereas the original isolate has eight. The results are
shown.

Original isolate:
... ACCCCC CCC ACT CAA ATT GAA CCT AGC ...
... Thr Pro Pro THr Gin lie Glu Pro Ser ...

Subsequent isolates:
... ACCCCC CCC CAC TCA AAT TGA ACC TAG
... Thr Pro Pro His Ser Asn STOP

Which of the following mechanisms best explains this genetic change?

® A ) Crossing over
2 B ) DNA excision repair
C ) Slipped-strand mispairing
D ) Thymidine dimer formation
E ) Transposon insertion
Correct Answer: C.

30. A 36-year-old man comes to the physician’s office without an appointment and demands to be seen immediately. He has had a sore throat for
24 hours and explains his problem is more important than the other patients' because he is a vocalist. He is extremely upset when the receptionist
explains he will need to wait for 1 hour to see the physician. He talks with her constantly while he waits. He says, "My wife and I split like 3 months
ago. We have a 6-month-old boy, but she was just so into her pregnancy, she had no time for me. And she doesn't really appreciate my music. I
need full support from my lady. Especially when my band's just about to make it big." Which of the following is the most likely personality disorder?

O A ) Antisocial
D B ) Borderline
O C ) Compulsive
0 D ) Dependent
E ) Histrionic
® F ) Narcissistic
O G ) Paranoid
0 H ) Schizoid
Correct Answer: F.
y
31. A 56-year-old man comes to the physician because of a 3-month history of intermittent regurgitation of material with an acidic taste. He is unable to
sleep and has missed 3 days of work during the past month because of the symptom. Various non-pharmacologic measures, including elevating the
head of his bed and decreasing his dietary fat intake and portion size, have resulted in only mild improvement. Physical examination shows no
abnormalities. Esophageal endoscopy shows severe erosions; there are no other structural abnormalities. The most appropriate pharmacotherapy
for this patient is a drug with which of the following mechanisms of action on the parietal cells?

® A) Agonism of cholecystokinin B (CCK 2) receptor


B ) Agonism of prostaglandin E receptor 3 (EP3) receptor
C ) Antagonism of histamine-2 (Hj) receptor
D ) Antagonism of muscarinic (M 3) acetylcholine receptor
3 E ) Irreversible inhibition of H +-K + ATPase
Correct Answer: E.

y
32. A 38-year-old man with a 3-year history of type 2 diabetes mellitus comes to the physician for a follow-up examination. In addition to taking an oral
antihyperglycemic agent, he has tried controlling his condition with diet modifications and exercise. He is 188 cm (6 ft 2 in) tall and weighs 113 kg
(250 lb); BMI is 32 kg/m 2. Physical examination shows no other abnormalities. His hemoglobin A 1cis 10%. The physician recommends initiation of
insulin injections to obtain better control over the patient's blood glucose concentration. The patient responds, "I know that insulin would help control
my blood sugar. But a lot of people in my family have diabetes, and insulin made them really sick at times." This patient is most likely at which of the
following stages of change regarding insulin administration?

O A ) Precontemplation
O B ) Contemplation
0 C ) Preparation
0 D ) Action
® E ) Maintenance
Correct Answer: B.
y
33. A 2-year-old boy has had recurrent bacterial infections with markedly increased absolute neutrophil counts in the blood (30,000-150,000/mm 3).
Studies of neutrophils show:
Test Substance Patient (% Control)
Bacterial killing
Normal opsonized patient serum 80± 20
Patient opsonized normal serum 70±15
Phagocytosis
Normal opsonized patient serum 85±20
Patient opsonized normal serum 60±20
Chemotaxis
Normal patient C5a 90±15
Patient patient C5a 10±5

Which of the following is the most likely diagnosis?

(• A ) Chronic granulomatous disease

B ) Leukocyte adhesion deficiency


C ) Myeloperoxidase deficiency
D ) NADPH oxidase deficiency
E ) Neutrophil-specific granule deficiency
Correct Answer: B.

y
34. A 62-year-old man has had petechiae and easy bruising for the past month. He has a 2-year history of alcoholic cirrhosis with portal hypertension.
Laboratory studies show:
Hemoglobin 10 g/dL
Hematocrit 30%
Leukocyte count 4000/mm 3
Neutrophils 65%
Lymphocytes 30%
Monocytes 4%
Eosinophils 1%
Basophils 0%
Reticulocyte count 8% of red cells
Mean corpuscular volume 102 pm 3
Platelet count 68,000/mm 3
Prothrombin time 16 sec
Partial thromboplastin time
28 sec
(activated)
Serum
Fibrinogen 200 mg/dL (N=200-400 mg/dL)
Fibrin split products <10 (N < 10)

Which of the following is the most likely cause of the pancytopenia?

C A ) Disseminated intravascular coagulation


O B ) Hypersplenism
® C ) Iron deficiency
,
D ) Vitamin B (thiamine) deficiency
O E ) Vitamin C deficiency
Correct Answer: B.
Y
35. A 35-year-old woman comes to the emergency department 3 hours after the sudden onset of severe right-sided back pain. She has a history of
recurrent urinary tract infections. Her most recent infection was 1 month ago; she was successfully treated for Klebsiella pneumoniae with
antibiotics. The patient is not taking any medications. She says that she has been eating more fast food and drinking less water during the past
month because of a busy schedule. She is sexually active and uses a spermicidal cream for contraception. Her temperature is 37°C (98.6°F), pulse
is 100/min, respirations are 14/min, and blood pressure is 138/66 mm Hg. Physical examination shows marked tenderness to palpation over the
right flank. Urinalysis shows:
pH 8
Protein 1+
RBC 30-35/hpf
WBC 20-25/hpf

An excess of which of the following is the most likely underlying cause of this patient's flank pain?

G A ) Calcium oxalate
B ) Cystine
G C ) Magnesium ammonium phosphate
@ D ) Potassium citrate
G E ) Uric acid
Correct Answer: C.

y
36. Following blockage of a cerebral artery, the neurons that die from hypoxia release a neurotransmitter that promotes excessive calcium influx, lysis,
and cell death in nearby undamaged neurons. Which of the following substances is the primary cause of this cascade of excitotoxicity?

® A ) Acetylcholine
B ) y-Aminobutyric acid
O C ) Glutamate
D ) Glycine
E ) Norepinephrine
Correct Answer: C.

Y 37
. A study is conducted to assess the relationship between angiotensin gene polymorphisms (I versus D alleles) and the development of progressive
renal disease due to IgA nephropathy. A total of 250 patients are enrolled: 125 patients with progressive renal disease due to IgA nephropathy
(Group X ) and 125 patients with IgA nephropathy with stable renal disease (Group Y). Results show that 67 patients in Group X have the D allele,
whereas 12 patients in Group Y have the D allele. Which of the following tests is most appropriate to determine whether there is a significant
difference in the prevalence of the D allele between these two groups?

O A ) Analysis of variance
O B ) Chi-square test
G C ) Linear regression
® D ) Nonparametric regression
O E ) Student's f-test
Correct Answer: B.

Y 38
. A 44-year-old woman comes to the physician because of a 3-week history of severe headache. She is 157 cm (5 ft 2 in) tall and weighs
86 kg (190 lb); BMI is 35 kg/m 2. Funduscopic examination shows papilledema. An MRI of the brain shows no abnormalities. A lumbar puncture is
done. Cerebrospinal fluid (CSF) analysis shows an opening pressure of 32 cm H 20. In addition to recommendations regarding dietary changes and
weight loss, treatment with acetazolamide is begun. This drug will most likely improve the patient's symptoms by which of the following
mechanisms?

O A ) Decreased CSF production by the choroid plexus


® B ) Decreased venous pressure around the arachnoid granulations
C ) Increased permeability of tight junctions (zonulae occludentes) between endothelial cells
D ) Increased sodium permeability of the ependyma
E ) Increased water transport by astrocyte end-foot processes
Correct Answer: A.
y NONSMOKERS SMOKERS

5
£
I
5
i

Diastolic blood pressure Cholesterol quintile, Diastolic blood pressure Cholesterol quintile,
quintile, (mm Hg) (mg/dL) quintile, (mm Hg) (mg/dL)

39. The graphs show the combined effect of total serum cholesterol concentration and diastolic blood pressure on age-adjusted coronary artery disease
death rates for both smokers and nonsmokers. Two persons, X and Y, have the following data:
Smokes Diastolic Blood Pressure Total Serum Cholesterol
X no 72 mm Hg 180 mg/dL
Y yes 95 mm Hg 250 mg/dL

Compared with X, which of the following is the relative risk of dying from coronary artery disease for Y?

0 A) 2
® B) 6
3 C ) 12
5 D ) 30
0 E ) 55
Correct Answer: C.

y
40. A 32-year-old woman comes to the physician because of a 3-month history of fevers, weakness, and night
sweats. Physical examination shows enlarged axillary lymph nodes. A photomicrograph of tissue obtained
biopsy of an affected node is shown. A stain for acid-fast bacilli is negative. Which of the following is most
likely involved in the pathogenesis of these findings?

0 A ) Activation of alternate complement pathway


C B ) Streptococcal infection of the region drained by the lymph node
® C ) Type I (immediate) hypersensitivity reaction
0 D ) Type III (immune complex-mediated) hypersensitivity reaction
O E ) Type IV (delayed) hypersensitivity reaction

'

m
Correct Answer: E.

y
41. A previously healthy 42-year-old Asian woman is brought to the emergency department because of a 24-hour history of nausea, vomiting, and
progressive lethargy. She has smoked 1 pack of cigarettes daily for 25 years and drinks four glasses of wine daily. She uses high dose -
acetaminophen daily for headaches. She does not use illicit drugs. She is 155 cm (5 ft 1 in) tall and weighs 50 kg (110 lb); BMI is 21 kg/m 2. She is
responsive to painful stimuli. Initial laboratory studies show increased hepatic aminotransferase. Which of the following effects of alcohol most likely
contributed to this patient's condition?

A ) Decreased generation of A/-acetyl-p-benzoquinoneimine


B ) Increased glucuronidation
•)
( C ) Increased hepatic glutathione stores
D ) Increased sulfation
O E ) Induction of cytochrome P450
Correct Answer: E.
y
42. A 27-year-old man has hepatomegaly and mild jaundice. Serum alkaline phosphatase and y-glutamyltransferase activities are increased. Which of
the following properties of these enzymes allows them to be used to detect defects in bile flow?

0 A ) Present in bile ductules


B ) Present on the basal surface of hepatocytes
0 C ) Released from the gallbladder
(S D ) Required for bile salt synthesis
E ) Required for bilirubin conjugation
F ) Required for heme catabolism
Correct Answer: A.

y
43. A 68-year-old woman is brought to the emergency department 3 hours after falling onto her right wrist while
working in her yard. Examination of the right wrist shows swelling and ecchymoses. Passive flexion of the
right wrist exacerbates the pain. An x-ray of the right wrist is shown. Which of the following is the most
likely cause of these findings?

O A ) Avascular necrosis of the scaphoid


C B ) Avulsion of the ulnar styloid process
(• C ) Dislocation of the first proximal phalanx
D ) Disruption of the radioulnar joint
O E ) Fracture of the distal radius

Correct Answer: E.

y
44. During a genome project, an experimental animal is found to have a DNA sequence with similarities to a growth hormone receptor. Which of the
following peptide segments shown is the transmembrane segment?

A
Ala Leu HisAspAsnTyrLysPro Glu PheTyrAsnAspAsp SerTrpVal Glu Phe Glu

Leu Thr Glu Glu Ser Asp Glu Tyr Glu Leu Asp His Gin Lys Ser Leu Gly Ala Lys Asp
B
Asp Asp SerGlyArgThrSerCysGly CysGluAla CysThr SerLeuCysAla Glu Leu
C
Lys His Ala Ser Lys Gly SerThrArg HisThrGly Gin AlaAsnArgArgArg Lys His Leu
D
AspTyrAsnAspAspGIu lie Asp lie Asp Asp Thr Asp Glu Thr Glu Glu Ser Asp Thr
E
Leu Val Leu lie Phe Gly Met lie Gly Val Val lie Val Val LeuLeuVal Leu Leu lie Phe

Gly Asp Glu Lys Leu Ala Leu Leu Leu Asp Gin Pro Glu Ser LeuThrAlaAlaAla Gly
F
Lye Ser Gly Leu Ala Glu Gin Arg Leu Ser Cys Glu Thr Ser lie Pro Lys Asp Tyr Thr Ser

lie Phe Leu Tyr

O A)
® B)
C)
D)
E)
F)
Correct Answer: E.
y
45. A 20-year-old woman comes to the physician because her menstrual period is 1 week late. Menses had occurred at regular 28-day intervals. She
had unprotected sexual intercourse with her boyfriend 20 days ago. A urine pregnancy test result is positive. Which of the following best describes
the stage of development of the embryo at this time?

O A ) The cytotrophoblast is present, but the syncytiotrophoblast has not yet formed
I B ) Gastrulation is complete, but there are only two germ layers
O C ) The neural plate is present, but the neural tube is not yet complete
(5 D ) Placental development is complete, but the embryo is resistant to teratogens

O E ) The sclerotome cells have begun migrating, but the somites have not yet formed
Correct Answer: C.

y
46. A 50-year-old man collapses and dies while running to catch an airplane. A photograph of his aortic valve is
shown. Which of the following best describes the lesion?

O A ) Calcified congenital bicuspid valve


B ) Infective endocarditis
(•: C ) Myxomatous degeneration
D ) Postinflammatory valve stenosis

Correct Answer: A.

y
47. A 16-year-old boy has a 6-month history of type 1 diabetes mellitus and requires daily insulin injections. His blood glucose control has been poor
-
recently, which the physician suspects may be related to the patient's reluctance to self administer insulin. Which of the following questions is most
appropriate to broach this issue with the patient?

O A ) "Giving yourself insulin injections can be difficult. What's it like for you?"
3 B ) "I get the feeling you have not been taking your insulin regularly. Would you like me to arrange some training for you by our nurses?"
C ) "It's been my experience that most poorly controlled diabetics have trouble giving themselves insulin. Do you have this problem?"
D ) "You are clearly having difficulty with insulin injections. Would you like to arrange for a family member to give you your shots?"
E ) "You seem to have missed your shots. How often has this happened?"
Correct Answer: A.

y
48. A 36-year-old man who is heterozygous for an LDL-receptor deficiency has a total serum cholesterol concentration of 330 mg/dL. After taking a drug
that inhibits cholesterol synthesis, his total serum cholesterol concentration decreases to 200 mg/dL. Which of the following proteins would be
upregulated as a result of drug therapy?

C A ) Cholesterol acyltransferase
B ) Hepatic lipase
C ) LDL receptor
(s D ) Lipoprotein lipase

E ) Lysosomal cholesterol esterase


Correct Answer: C.

y
49. A study is conducted to evaluate the effectiveness of cough medication in children. A total of 120 patients under the age of 12 years who have been
diagnosed with a viral upper respiratory tract infection are randomly selected to receive dextromethorphan or no treatment. The participants were
recruited from several emergency departments in an urban area. Parents complete a rating scale for their children's cough at baseline and 1 day
after receiving dextromethorphan or nothing. The results show a statistically significant improvement in cough scores for patients receiving
dextromethorphan (p=0.04) compared with children receiving no treatment. Which of the following is the most significant limitation to the internal
validity of this study?

A ) Lack of blinding between groups


B ) Limited number of participants
@ C ) Marginal statistical significance
D ) Study location limited to only emergency departments
E ) Subjective nature of the survey instrument
Correct Answer: A.
y
50. A 56-year-old man comes to the physician for a routine examination. His stool is positive for occult blood. Colonoscopy shows a pedunculated
growth that is coarsely lobulated and 1.4 cm in diameter. The excised lesion is covered by colonic epithelium and composed of branching glands
lined by tall hyperchromatic cells. Some areas appear mildly dysplastic, but there is no evidence of invasion of the stalk. Which of the following
oncogenes is most likely to be activated in this lesion?

A ) erb
B ) fos
O C ) K-ras
® D ) p53 ( TP53 )
D E ) Rb
F ) src
Correct Answer: C.

PART 4

Y 1. A 59-year-old man comes to the physician because of a 3-month history of progressive difficulty swallowing; he has had a 6.8-kg (15-lb) weight loss
during this period. He has smoked 1 pack of cigarettes daily for 40 years and consumes four to six beers daily. He is cachectic. Bilateral wheezes are
heard on auscultation of the chest. A chest x-ray shows enlargement of the paratracheal and hilar lymph nodes. Subsequent endoscopy shows a
6-cm mass in the mid esophagus. The mass is centrally ulcerated with an elevation of the surrounding mucosal rim. Examination of a biopsy
specimen of the mass is most likely to show neoplastic cells with which of the following?

O A ) Acinar formation
O B ) Foci of keratinization
® C ) Intracytoplasmic pigment
2 D ) Neuroendocrine granules
E ) Smooth muscle differentiation
Correct Answer: B.

2. A 52-year-old man comes to the physician because of a 5-day history of headaches and muscle weakness. His blood pressure is 175/95 mm Hg.
Physical examination shows no other abnormalities. Laboratory studies show:
Serum
Na + 142 mEq/L
K* 2 mEq/L
HCO 3- 33 mEq/L
Creatinine 1.4 mg/dL
Cortisol <2 pg/dL
Arterial blood gas analysis on room air:
pH 7.45
Pco 2 48 mm Hg

Plasma renin activity is undetectable. Which of the following is the most likely cause of this patient's hypertension?

O A ) Addison disease
0 B ) Chronic kidney disease
0 C ) Cushing disease
® D ) Hyperaldosteronism
0 E ) Pheochromocytoma
Correct Answer: D.

y
3. A 21-year-old man comes to the physician because of a lump in his scrotum for 3 months. Physical examination shows a 1-cm nontender nodule in
the right scrotum. Transillumination shows a solid, noncystic structure. An operation to remove the testis is done. After incision of the skin and
subcutaneous tissue, the next fascial layer to be incised in this patient is a direct extension of which of the following abdominal layers?

O A ) Dartos muscle
C B ) External abdominal oblique muscle
C ) Internal abdominal oblique muscle
($ D ) Peritoneum

0 E ) Transversalis fascia
Correct Answer: B.
y
4. A 24-year-old man who is serving overseas in the US Armed Forces is brought to a medical unit 30 minutes after sustaining multiple injuries caused
by an explosion that forcibly ejected him from the vehicle he was driving. On arrival, he is conscious and hemodynamically stable. His respirations
are 30/min. Physical examination shows visible shrapnel protruding from the lower sternum. Which of the following structures within the pericardial
sac is most likely damaged in this patient?

0 A ) Ascending aorta
3 B ) Left atrium
0 C ) Left ventricle
G D ) Right atrium
0 E ) Right ventricle
Correct Answer: E.

5. A 60-year-old man develops palpitations over a 6-week period during which he has been drinking approximately 8 cups of coffee per day. A rhythm
strip from his ECG is shown. Which of the following is most likely responsible for the palpitations in this man?

C A ) Second-degree atrioventricular block


B ) Sinoatrial exit block
O C ) Supraventricular premature beats
@ D ) Third-degree atrioventricular block
E ) Ventricular tachycardia
Correct Answer: C.

y
6. A 42-year-old woman comes to the physician because of a 3-week history of numbness of the fingers on the left hand. Neurologic examination shows
loss of touch graphesthesia and loss of two-point discrimination in the left hand. This patient most likely has a lesion in which of the following
locations of the hemispheres shown?

O A)
B)
C)
3 D)
3 E)
® F)
3 G)
3 H)
I)
3 J)
Correct Answer: B.
y
7. The graph shows the relationship between serum leptin concentration and BMI, expressed as ( weight in / <
kg) / (height in meters) 2 for eight patients labeled A through H. The dashed lines indicate the upper and
lower limits of normal. Which of the following patients most likely has primary leptin deficiency?
2
A B c
I
O A ) Patient A 8
2 D Normal E
B ) Patient B
i
3 C ) Patient C r

D ) Patient D
2
Ir
F G H
E ) Patient E 2
>
BMI (kg/m )
® F ) Patient F
3 G ) Patient G
3 H ) Patient H
Correct Answer: H.
/
8. A 68-year-old woman with a 20-year history of hypertension is brought to the emergency department 30 minutes after the sudden onset of right-sided
weakness. Her speech is dysarthric. Physical examination shows drooping of the right side of the face. Visual field testing shows no deficits. Muscle
strength is 4/5 in the right upper extremity and 3/ 5 in the right lower extremity. Deep tendon reflexes are 2+ in the upper extremities and left lower
extremity and 3+ in the right lower extremity. Sensation is intact throughout. Babinski sign is present on the right. Mental status examination shows no
abnormalities. Which of the following is the most likely cause of this patient's condition?

O A ) Border zone hypoperfusion


B ) Embolism of the right anterior cerebral artery
O C ) Lacunar infarct of the internal capsule

( ) D ) Ruptured aneurysm of the circle of Willis

E ) Thrombus of the left posterior cerebral artery


Correct Answer: C.

9. A 50-year-old man is brought to the physician 2 days after the onset of double vision and drowsiness. Three weeks ago, he began using over-the-
counter cimetidine for dyspepsia. He has epilepsy treated with phenytoin for the past 2 years. Physical examination shows nystagmus and an ataxic
gait. Which of the following effects of cimetidine on phenytoin is the most likely cause of the adverse effects in this patient?

0 A ) Altered binding to plasma proteins


B ) Altered binding to tissue receptors
3 C ) Altered distribution to active receptors
® D ) Inhibition of metabolism
E ) Inhibition of renal excretion
Correct Answer: D.

10. A 60-year-old woman dies of a myocardial infarction. Incidental findings on examination at autopsy include multiple 0.5- to 1-cm stool-filled
outpouchings of the sigmoid colon alongside the taeniae coli. The outpouchings protrude through the muscularis propria. Microscopic examination
of a lesion shows a thin wall consisting of attenuated colonic mucosa and submucosa. Which of the following mechanisms best explains the
formation of these lesions in the colon?

O A ) Abnormal development of the muscularis propria


B ) Atrophy of the muscularis propria
C ) Hyperplasia of the columnar epithelium
® D ) Increased intraluminal pressure
E ) Neoplastic proliferation of the columnar epithelium
Correct Answer: D.

* i i. A previously healthy 32-year-old man is brought to the emergency department because of a 1-day history of intermittent flashing spots and blurred
vision, vomiting, confusion, and difficulty walking. He is a painter and says that yesterday he cleaned up a spilled bottle of paint thinner.
Ophthalmologic examination shows dilated pupils with hyperemia of the optic disc and retinal edema. Physical examination shows tachypnea.
Serum studies show an anion gap metabolic acidosis. The most appropriate pharmacotherapy for this patient is most likely to inhibit the activity of
which of the following?

O A ) Alcohol dehydrogenase
0 B ) Aldehyde dehydrogenase
0 C ) Formaldehyde dehydrogenase
® D ) Lactate dehydrogenase
3 E ) Pyruvate dehydrogenase
Cnrrorf Ancu/or A

y
12. A 35-year-old homeless man is brought to the emergency department by police after they found him slumped on a bench. His pulse is 40/min,
respirations are 10/min, and blood pressure is 100/60 mm Hg. He is stuporous but can be aroused when shaken mildly. Physical examination shows
constricted pupils. This patient is most likely abusing which of the following substances?

0 A ) Alcohol
3 B ) Barbiturates
C ) Benzodiazepines
(5 D ) Marijuana

E ) Opioids
Onrrfirt Answer F
13. In a 3-week-old knockout mouse, a leg bone reaches only half the length of that of a similar-aged wild-type mouse. The epiphyseal plates in the
bone of the transgenic animal are significantly narrower than in the bone of the wild-type animal. The knockout mouse most likely has a deficiency
of which of the following?

O A ) Calcitonin
3 B ) Cortisol
® C ) Insulin-like growth factor I
D ) Platelet-derived growth factor
E ) Thyroxine (T4)
Correct Answer: C.

y
14. A 14-year-old boy is brought to the physician's office because of decreased appetite and abdominal pain over the past 3 weeks. His mother says
that during the same time he has withdrawn from everyone and sleeps constantly. He describes his symptoms vaguely. Physical examination is
normal. During further history-taking, it is most critical for the physician to obtain information about which of the following?

0 A ) Developmental history
B ) Family history of affective disorders
0 C ) Orientation to time, place, and person
® D ) School history
C E ) Suicidal ideation or attempts
Correct Answer: E.

15. A 57-year-old man is brought to the emergency department because of a 2-hour histoiy of severe abdominal pain and vomiting. Abdominal
examination shows diffuse tenderness. A CT scan of the abdomen and abdominal radiograph are shown; the arrows indicate an abnormality. The
most likely cause of these findings is perforation of which of the following structures?

0 A ) Duodenum
® B ) Esophagus
C ) Ileum
3 D ) Stomach
3 E ) Transverse colon
Correct Answer: A.
y

-
16. A 57-year old man has a hemoglobin concentration of 18.5 g/dL. A peripheral blood smear is shown. These
findings are most consistent with which of the following disorders? O
0 A ) Agnogenic myeloid metaplasia l
O

5Cc %
B ) Chronic obstructive pulmonary disease
3 C ) Hereditary hemochromatosis
®

*
D ) Hypersplenism
E ) Myelodysplastic syndrome ) t
’Aral

Correct Answer: B.
McPJP O'S t '
-
y
17. A 3-year-old girl is brought to the physician because of a 1-year history of short stature. She has no history of major medical illness. She is below
the 3rd percentile for height and at the 10th percentile for weight. Physical examination shows coarse facial features and contractures of the large
joints. X-rays show dysostosis multiplex. Plasma lysosomal enzyme analysis shows increased p-hexosaminidase, p-glucuronidase, p-
galactosidase, and a-fucosidase activities. Which of the following mechanisms is the most likely cause of the lysosomal enzyme findings in this
patient?

A ) Abnormal targeting of these enzymes to lysosomes


B ) Deficiency of the other lysosomal enzymes in the cytoplasm
0 C ) Degradation of these enzymes within the cytoplasm
D ) Degradation of these enzymes within the lysosomes
E ) Storage of these enzymes within the cytoplasm
Correct Answer: A.

. .
18 A 55-year-old man comes to the office because of a 7-year history of red, dry, itchy skin over his back and buttocks Use of emollients and topical corticosteroids has not
improved his symptoms. He has no other history of major medical illness. His vital signs are within normal limits. Physical examination shows the findings in the photograph.
Laboratory studies show eosinophilia and increased serum concentrations of IgA and IgE. Malignant transformation of which of the following cell types is the most likely cause
of the findings in this patient?

O A ) Blastocytes
O B ) Eosinophils
• C ) Macrophages
O D ) Mastoid cells
O E ) Monocytes
O F ) T lymphocytes
Correct Answer: F.

y
19. A 40-year-old woman comes to the physician because of a 2-month history of increasing redness around her nose and cheeks, and pimples around
her mouth. She appears very upset and tells the physician, "I look like a teenaged Santa Claus." Which of the following initial responses by the
physician is most appropriate?

0 A ) "Can you tell me how often you go out in the sun and what kind of protection you use?"
B ) "Don't worry. Let me assure you this is not a serious condition."
O C ) "It's upsetting when you have a skin problem on your face."
(
i D ) "Maybe a little. At least you are keeping your sense of humor about the situation."
2 E ) "You really do look rather odd. I can understand why you are so upset."
Correct Answer: C.
y
20. A 15-year-old girl with acute myelogenous leukemia undergoes a bone marrow transplant procedure. Her 19-year-old brother is the donor. Blood
group and human leukocyte antigen (HLA) typing of patient and donor leukocytes are:
Patient Donor
Blood Group -
AB, Rh positive -
A, Rh positive
HLA-A 2, 3 2, 3
HLA-B 5, 7 5, 19
HLA-DR 2, 5 2, 5

Three weeks later, she develops a temperature of 37.4°C (99.3°F); a diffuse, erythematous rash over most of the body, including the palmar and
-
plantar surfaces; and watery diarrhea. Serum studies show an AST activity of 120 U/L (N=5-55) and ALT activity of 180 U/L (N=5 45). Which of the
following is the most likely diagnosis?

0 A ) Acute rejection
B ) Cytomegalovirus infection
C ) Graft-versus-host disease
D ) Hepatitis C
5
( E ) Hyperacute rejection
0 F ) Leukemia recurrence
0 G ) Serum sickness
Correct Answer: C.
y
21. A 26-year-old man comes to the physician because of a 1-week history of temperatures to 38.3°C (101°F), chills, and chest and back pain. Two
weeks ago, he returned from a trip to Costa Rica, where he had been performing ecologic studies of bats in their cave habitats. He reports exposure
to bat urine and feces during his studies. Today, his temperature is 38.1°C (100.6°F). Physical examination shows no other abnormalities. A
complete blood count and serum chemistry studies are within the reference ranges. A chest x-ray shows bilateral reticulonodular infiltrates. The
virulence of the most likely infectious agent in this patient depends on which of the following?

A ) Elaboration of polyribitol phosphate capsule


B ) Growth inside nonactivated macrophages
C ) Invasion of host tissues and vasculature
(5 D ) Phenotypic switching from yeast to hyphae in the host

E ) Production of a lipopolysaccharide capsule


Correct Answer: B.

y
22. A 36-year-old man comes to the physician after he noticed a dark lesion on his back. Physical examination shows a dark, irregularly shaped, raised
region on the back. He has a history of frequent sun exposure at the beach. A biopsy specimen of the region confirms the diagnosis of a melanoma.
Which of the following embryonic tissues most likely formed this tumor?

O A ) Dermatome
C B ) Lateral plate mesoderm
O C ) Neural crest
D ) Nonneural ectoderm
E ) Prechordal plate mesoderm
Correct Answer: C.

23. One hundred patients discharged from the hospital with a diagnosis of proximal deep venous thrombosis were followed for the development of
venous ulceration for up to 8 years ( DVT group). They were compared to a group of 200 patients who were discharged from the hospital with a
diagnosis of pneumonia (the control group). At the end of 5 years, 25 patients in the DVT group and 10 patients in the control group developed
venous ulceration. Which of the following is the attributable (excess) risk per one hundred patients for development of venous ulceration following
deep venous thrombosis over 5 years?

O A) 1
D B) 2
0 C ) 2.4
0 D ) 2.5
E ) 10
® F ) 20
0 G ) 24
0 H ) 25
Correct Answer: F.
y
24. A 25-year-old primigravid woman at 12 weeks' gestation comes to the physician 1 hour after she passed a small amount of tissue vaginally at home.
Gross examination of this tissue shows an obvious fetus. Microscopic examination of tissue obtained via dilatation and curettage shows chorionic
villi with focal edema and trophoblastic proliferation. Which of the following is the most likely diagnosis?

O A ) Choriocarcinoma
B ) Complete hydatidiform mole
C ) Partial hydatidiform mole
® D ) Placenta accreta
E ) Placental site trophoblastic tumor
Correct Answer: C.

y
25. A 35-year-old woman with sickle cell trait is brought to the emergency department because of severe left flank pain for 1 hour. She has a history of
frequent headaches. Her blood pressure is 150/90 mm Hg. Pressure on the costophrenic angle causes pain. Urinalysis shows a few RBCs and
numerous neutrophils. A CT scan of the abdomen shows obstruction and dilation of the left ureter and blunting of several renal papillary pyramids.
Which of the following is the most likely diagnosis?

C A) Acute glomerulonephritis
C B) Acute tubular necrosis
C ) Diabetic nephropathy
(5 D ) Papillary transitional cell carcinoma
E ) Renal papillary necrosis
Correct Answer: E.

26. A 22-year-old man is brought to the emergency department 1 hour after being involved in a motor vehicle collision. Physical examination shows
deep lacerations to the right lower extremity. Which of the following functional capabilities of the involved muscles will be decreased by fibrotic scar
formation at the sites of injury?

O A ) Formation of glycogen deposits


O B ) Maintenance of prolonged tension
® C ) Passive stretching
3 D ) Relaxation after contraction
Correct Answer: C.

y
27. A 68-year-old man with dementia, Alzheimer type, has improvement in memory and cognition with donepezil therapy. Which of the following is the
most likely mechanism of this beneficial effect?

C A ) Blockade of choline reuptake into presynaptic terminals


3 B ) Blockade of effects on the nicotinic receptors on muscle
0 C ) Enhanced influx of calcium into the presynaptic terminal
(• D ) Prevention of acetylcholine release from subsequent action potentials
C E ) Prolongation of acetylcholine effects in the synaptic cleft
Correct Answer: E.

y
28. A 21/2-year-old boy is admitted to the hospital because of generalized weakness, repeated episodes of vomiting, and coma after several days of
reduced food intake because of a minor febrile illness. He is treated successfully with intravenous glucose. After 13 hours of fasting when the child
is well, his serum glucose, p-hydroxybutyrate, and free carnitine concentrations decrease markedly, and serum concentrations of long-chain
acylcarnitines increase. Triglycerides containing only medium-chain fatty acids are fed, and the serum p-hydroxybutyrate concentration increases to
the reference range. Which of the following disorders of fatty acid metabolism is the most likely diagnosis?

C A ) Carnitine-acylcarnitine translocase deficiency


B ) Dietary carnitine deficiency
C ) Fatty acyl CoA synthetase deficiency
@ D ) Medium-chain acyl CoA dehydrogenase deficiency
E ) Short-chain acyl CoA dehydrogenase deficiency
Correct Answer: A.
29. An investigator is conducting a study of the efficacy of certain vitamins in helping to prevent heart disease in women. Eight thousand women,
45 years of age or older, are enrolled in the study. Two thousand women receive vitamin C only, 2000 receive vitamin E only, 2000 receive both
vitamins C and E, and 2000 receive a placebo. The investigator finds that the vitamins are ineffective in helping to prevent heart disease. Which of
the following best describes the study design used in this clinical trial?

O A ) Crossover
B ) Cross-sectional
® C ) Factorial
D ) Nested
O E ) Simple
Correct Answer: C.

30. An 8-year-old girl is brought to the physician by her mother 1 hour after her cat scratched her on her right arm. The mother says that the patient
developed bright red streaks along her arm within minutes of being scratched. The scratches were painful, and the patient was crying when she
showed the injury to her mother. Examination of the right forearm shows the surface linear streaks with intact skin, but there is no blood. The
appearance of the red streaks most likely indicated which of the following host responses to this injury?

O A ) Bacteria-generated capillary congestion


B ) Complement-activated coagulation
® C ) Histamine-stimulated vasodilation
-
D ) IgE mediated eosinophil degranulation
E ) Trauma-induced arteriolar hemorrhage
Correct Answer: C.
y
31. A 28-year-old woman, gravida 3, para 2, comes to the physician to request an elective abortion 2 days after she received a positive home
pregnancy test result. Her last menstrual period was 6 weeks ago. Ultrasonography shows a uterus consistent in size with a 6-week gestation. After
counseling, she decides to undergo a medication-induced abortion rather than a surgical abortion. An oral medication is administered, and she is
instructed to return to the office in 2 days to receive a second medication. The first drug administered most likely belongs to which of the following
classes?

O A ) Human chorionic gonadotropin antibodies


B ) Oxytocics
O C ) Progesterone antagonists
(? D ) Prostaglandin synthesis inhibitors
E ) Selective estrogen receptor modulators
Correct Answer: C.
y
32. A 35-year-old woman is brought to the emergency department comatose after she sustained multiple injuries j
in a motor vehicle collision. During the next few days, she develops sepsis and respiratory insufficiency, and ‘
\
then she dies. A photomicrograph of her lungs obtained at autopsy is shown. If this patient had recovered,
v;* &
»
regeneration of the alveolar epithelium would have been accomplished via hyperplasia of which of the
following cell types?
,3$
O A ) Alveolar capillary endothelial cell )‘JS $ }
>
B Aiveoiar macrophage
O C ) Chondrocyte ftVfcSgkj,
O D ) Ciliated columnar epithelial cell
E ) Club cell ^
3 F ) Goblet cell
® G ) Kulchitsky cell ^ ,
H ) Squamous epithelial cell .
3 I ) Type I pneumocyte
O J ) Type II pneumocyte

Correct Answer: J.

33. A researcher in a pharmaceutical company designs a new protease inhibitor that inhibits replication of HIV in T lymphocytes in culture. In
subsequent assays, which of the following findings is most likely to indicate that the compound is working specifically as a protease inhibitor?

O A ) The drug prevents integration of proviral DNA into the host genome
B ) The RNAis partially reverse transcribed into proviral DNA
® C ) There is a lack of a mature core
D ) Transcription from the HIV promoter is blocked
E ) The virus does not bind to CD4 in the presence of the drug
Correct Answer: C.
y

34. A 1-year-old boy is brought to the emergency department because of a 6-day history of temperatures to 39.4°C (103°F) and a 2-day history of a
severe diaper rash and swelling of his hands and feet. His temperature is now 39.4°C (103°F). A photograph of the genital area is shown. The face
and lips appear red and the conjunctivae appear injected. There is bilateral cervical lymphadenopathy. The lungs are clear to auscultation. Cardiac
examination shows an S3 gallop with no murmur. There is edema and erythema of the hands and feet. Which of the following pathologic findings is
most likely in this patient?

O A ) Acute arteritis with aneurysms in coronary arteries


B ) Epidermal hyperplasia with epidermal microabscesses and parakeratosis
® C ) Granulomas with caseous necrosis in cervical lymph nodes
0 D ) Granulomatous arteritis in cervical and temporal arteries
E ) Paracortical lymphoid hyperplasia with eosinophilic intranuclear inclusions in perihilar lymph nodes
Correct Answer: A.

y
35. A healthy 21-year-old woman is concerned that her son will develop a familial neurologic disorder. Her older brother died at the age of 45 years after
developing restlessness, forgetfulness, incoordination, altered handwriting and speech, and subsequently, chorea. Autopsy showed atrophy of the
basal ganglia. Their mother died at the age of 55 years of a similar disorder. Which of the following best approximates the risk of this woman's son
for developing the disorder?

0 A ) Essentially zero
B ) 25%
0 C ) 33%
® D ) 50%
0 E ) 100%
Correct Answer: B.

y
36. A 55-year-old man comes to the physician because of a 2-month history of hoarseness and difficulty swallowing. He underwent an operation to resect a lobe of his lung
because of lung cancer 1 year ago. Physical examination shows ptosis of the left eyelid and miosis of the left pupil. There is diminished flushing and sweating on the left half of
the face. This patient most likely has disordered function of neurons at which of the following labeled locations in the photograph of a cross section of the spinal cord?

A)
O B)
* C)
O D)
O E)
O F)
Correct Answer: B.
y
37. A healthy 28-year-old woman participates in an exercise study. Several physiologic variables are measured as she runs on a
treadmill. The ambient room temperature is 75°F. She reaches a steady state that increases her oxygen consumption threefold.
She continues to exercise at that level for 20 minutes. During the first 5 minutes of exercise, the vascular resistance of which of
the following is likely to show the greatest increase from resting values in this volunteer?

A ) Coronary
B ) Cutaneous
® C ) Exercising muscle
D ) Pulmonary
O E ) Splanchnic
F ) Systemic
Correct Answer: E.

y
38. A 31-year-old woman is brought to the emergency department by her husband because of difficulty breathing and severe
muscle weakness for 10 minutes. She has a 3-year history of myasthenia gravis treated with neostigmine. The husband reports
that she doubled her dosage 2 days ago because she was feeling extraordinarily weak, but her weakness has increased since
then. Which of the following events is the most likely cause of the increased muscle weakness in this patient?

A ) Autoimmune hyperactivation of nicotinic receptors


B ) Autoimmune inactivation of muscarinic receptors
O C ) Desensitization of nicotinic receptors
D ) Excessive degradation of acetylcholine
® E ) Hypersensitization of muscarinic receptors
F ) Insufficient release of acetylcholine
Correct Answer: C.

y
39. A 62-year-old man with hypertension comes to the physician because of a 10-day history of fatigue and shortness of breath.
His temperature is 37.4°C (99.4°F), pulse is 102/min, respirations are 18/min, and blood pressure is 110/70 mm Hg. Pulmonary
examination shows diffusely decreased breath sounds and a prolonged expiratory phase. Cardiac examination shows distant
heart sounds. There is 1+ pretibial edema of the lower extremities. Pulmonary function testing shows:
FEV , 65% of predicted
FVC 90% of predicted
FEV /FVC 60% of predicted

Which of the following is the most likely diagnosis?

A ) Cardiac tamponade
B ) Chronic obstructive pulmonary disease
® C ) Congestive heart failure
D ) Mitral stenosis
E ) Pleural effusion
F ) Sarcoidosis
Correct Answer: B.
40. A 55-year-old man has a cough, hemoptysis, and dyspnea. He has smoked 2 packs of cigarettes daily for the past 30 years.
Examination shows bilateral conjunctival edema, distention of sublingual veins, and edema of the upper extremities. Which of
the following additional findings is most likely to be present on examination?

A ) Bisferiens carotid pulsation


0 B ) Increased jugular venous pressure
C ) Pansystolic apical murmur
O D ) S3
O E ) S4
Correct Answer: B.

y
41. A study is conducted to assess the prevalence and incidence of obesity in a group of middle-aged men. A total of 900 men
between the ages of 40 and 50 years are chosen to participate. The table shows the number of participants at baseline and at
the follow-up study 5 years later for each BMI range.
Number of Participants
BMI (kg/m 2) Baseline 5 Years Later
Normal (< 25) 300 225
Overweight (between 25 and 30) 500 525
Obese (> 30) 100 150

The prevalence of obesity in this study population after 5 years is which of the following?

A ) 50/100
B ) 50/500
C ) 50/800
D ) 50/900
E ) 150/375
0 F ) 150/525
G ) 150/750
O H ) 150/900
Correct Answer: H.

42. A 53-year-old man comes to the physician for a follow-up examination. He began antibiotic therapy 1 month ago for acute
bacterial pneumonia. At the time of diagnosis, a chest x-ray showed patchy pneumonic consolidation and a 2.5-cm peripheral
nodular lesion in the right upper lobe. His symptoms have since resolved, and an x-ray today shows absence of the
pneumonic consolidation. However, the lesion is still present. A PPD skin test is nonreactive. A wedge resection is done by
thoracoscopy. Histologic examination of the resected tissue shows a solid, well-circumscribed nodule composed
predominantly of mature cartilage admixed with small amounts of fibrous tissue and occasional clefts lined by benign
respiratory epithelium. Which of the following best describes this patient's pulmonary lesion?

A ) Abscess
B ) Adenoma
C ) Choristoma
0 D ) Hamartoma
E ) Teratoma
Correct Answer: D.
y
43. A 40-year-old woman has had hypercalcemia for 1 year and recently passed a renal calculus. Serum parathyroid hormone and
calcium concentrations are increased, and serum phosphate concentration is decreased. Parathyroid hormone most likely
causes an increase in the serum calcium concentration by which of the following mechanisms?

A ) Decreased degradation of 25-hydroxycholecalciferol


B ) Direct action on intestine to increase calcium absorption
C ) Direct action on intestine to increase magnesium absorption
® D ) Increased synthesis of 25-hydroxycholecalciferol
E ) Inhibition of calcitonin production
O F ) Stimulation of 1,25-dihydroxycholecalciferol production
Correct Answer: F.

y
44. Which of the following best explains why deoxygenated blood can carry more carbon dioxide for a given Pco 2 than oxygenated
blood?

A ) Deoxyhemoglobin does not bind to 2,3-bisphosphoglycerate as efficiently as oxyhemoglobin


B ) Deoxyhemoglobin has a lower capacity to form carbamino compounds than oxyhemoglobin
® C ) Deoxyhemoglobin has a lower pKa than oxyhemoglobin
O D ) Deoxyhemoglobin is a better buffer of hydrogen ions than oxyhemoglobin
E ) Oxygen and carbon dioxide compete for the same binding site in hemoglobin
F ) Oxyhemoglobin binds nitric oxide with a higher affinity than deoxyhemoglobin
Correct Answer: D.

y
45. A 17-year-old boy with chronic renal insufficiency undergoes a
unilateral nephrectomy. A photograph of the resected kidney is
shown. Which of the following pathologic processes is most likely
present in the kidney?

A ) Acute glomerulonephritis
B ) Acute tubular necrosis
C ) Angiomyolipoma
D ) Autosomal recessive polycystic kidney disease
E ) Fibromuscular dysplasia
O F ) Hydronephrosis
O G ) Nephroblastoma SPECIMEN

O H ) Papillary necrosis
® I ) Renal vein thrombosis
J ) Staghorn calculus
Correct Answer: F.

y
46. A 25-year-old woman at 38 weeks' gestation undergoes an oxytocin stress test to evaluate the integrity of the placental
circulation. Based on the results, a full-scale induction of labor with oxytocin is started. Treatment with oxytocin is most likely to
activate which of the following substances in this patient's uterine smooth muscle?

A ) Adrenergic nerve terminals


B ) Cholinergic nerve terminals
C ) Ligand-gated calcium channels
® D ) Na *-K * ATPase
E ) Voltage-gated chloride channels
Correct Answer: C.
y
47. A 73-year-old man with polymicrobial sepsis begins treatment with a variety of antibiotics, including gentamicin (current dose
indicated in table). Three days later, serum gentamicin concentrations are 8.8 pg/mL (peak ) and 2.5 pg/mL (trough). Which of
the following dose regimens is most appropriate for achieving a peak serum gentamicin concentration between 4 and 10 pg/mL
and a trough concentration of less than 2 pg/mL?

Dosage
Intravenous Administration (mg) Time Between Each Dose (hours)
(Current = 80 mg ) (Current = 8 hours)
A) 60 6
B) 80 6
C) 80 12
® D) 120 8
E) 120 12
Correct Answer: C.

y
48. In an experimental design evaluating assays for the bactericidal effect of inflammatory cells, which of the following compounds
is considered the most efficient in the killing of bacteria by neutrophils?

A ) Bactericidal permeability increasing protein


O B ) Hypochlorite
O C ) Lactoferrin
® D ) Lysozyme
E ) Major basic protein
Correct Answer: B.

y
49. An 80-year-old man comes to the physician because of bleeding gums and bruises under his skin. He lives by himself and
follows a diet low in vegetables and fruits. His prothrombin time is normal. Which of the following enzymes is most likely
compromised in this clinical scenario?

A ) 2,3-Epoxide reductase
B ) Glucose 6-phosphate dehydrogenase
C ) Hexokinase
D ) Methionine synthase
O E ) Prolyl hydroxylase
® F ) Pyruvate carboxylase
G ) Pyruvate dehydrogenase
Correct Answer: E.

y
50. A 40-year-old woman has easy bruising and menorrhagia. Platelet count is 100,000/mm 3, and bleeding time is prolonged.
Factor VIII (antihemophilic factor) concentration is decreased, and platelet adhesion to collagen is abnormal. Which of the
following is the most likely diagnosis?

A ) Hemophilia A
B ) Hemophilia B
C ) Immune thrombocytopenic purpura
® D ) Ingestion of aspirin
E ) Systemic lupus erythematosus
F ) Thrombotic thrombocytopenic purpura
O G ) von Willebrand disease
Correct Answer: G.

You might also like